Bailey & Scott's 14th Edition Q and A

You might also like

Download as pdf or txt
Download as pdf or txt
You are on page 1of 111

CHAPTER 1 Microbial Taxonomy 3.

e1

Chapter Review
1. The most specific and exclusive taxon used in the classifi- 4. Which of the following is not a correct use of the bino-
cation of microorganisms is: mial nomenclature system? (Select all that apply.)
a. Family a. Staphylococcus Aureus
b. Order b. S. aureus
c. Species c. Staphylococcus aureus
d. Genus d. Staphylococcus aureus
2. The process consisting of a series of methods designed 5. Labeling: Label each of the following characteristics as
to provide the microbiologist with relevant and useful either a phenotypic (P) or a genotypic (G) characteristic.
clinical information about a microorganism is: _____ Color of growth on artificial media
a. Classification _____ The presence of an antibiotic-resistance DNA
b. Identification sequence
c. Organization _____ The shape of the bacterial cell
d. Taxonomy _____ The arrangement of the bacterial cells on a
3. Classification and naming of organisms is useful in diag- microscope slide
nostic microbiology for all of the following except: _____ The ability of the organism to ferment lactose
a. Providing standardized groupings for identification
b. Standardized groupings are always genotypically
similar at .98%
c. Standardized groupings share similar phenotypic
traits
d. The ability of organisms within a standard group may
be identified using similar methods
Tille: Bailey & Scott’s Diagnostic Microbiology, 14th Edition
Chapter 01: Microbial Taxonomy

Answers to Case Studies and Review Questions

Review Questions

Multiple Choice

1. c; 2. b; 3. b; 4. a; 5. P, G, P, P, P

Copyright © 2017, Elsevier Inc. All Rights Reserved.


CHAPTER 2 Bacterial Genetics, Metabolism, and Structure 23.e1

Chapter Review
1. The periplasmic space is required for: 7. Which chemical or physical property is essential for the
a. Nutrient collection in both gram-positive and gram- conservation of genetic information?
negative bacteria a. Complementation between base pairs
b. Collection and enzymatic degradation of nutrients in b. Double-stranded
gram-negative bacteria c. Antiparallel structure
c. Nutrient detoxification and enzymatic degradation in d. All are equally important
all bacteria 8. Expression of a biochemical molecule in an organism
d. None of the above requires:
2. Prokaryotic chromosomes: a. Replication only
a. Are double-stranded RNA molecules b. Transcription only
b. Are single-copy, double-stranded DNA molecules c. Transcription and translation of a protein
c. Are linear double-stranded DNA molecules d. All of the above
d. Are unable to replicate independently of plasmids 9. True or False
3. Bacterial cells genetically evolve by: _____ All bacteria are considered competent.
a. Recombination with plasmids, transposons, and other _____ Conjugation requires cell-to-cell contact.
bacterial chromosomes _____ Oxidative phosphorylation occurs across the
b. Mutation and recombination cell membrane in bacteria.
c. Use of the mechanisms of transduction, transformation, 10. Short Answer
and conjugation Provide an explanation for why bacteria are capable of
d. All of the above rapidly responding to changes in their environment
4. Transcription is the: based on molecular and cellular structure. Bacteria
a. Copying of DNA to RNA are prokaryotes; the genetic material is not contained
b. Changing of DNA to RNA within a nucleus, allowing replication transcription
c. Production of a complementary DNA and translation to occur simultaneously.
d. Completion of a protein sequence
5. A eukaryotic cell:
a. Is smaller and less complex than a prokaryotic cell
b. Is able to grow only in aerobic conditions
c. Contains membrane-bound organelles
d. Is unable to grow outside of another cell
6. Matching: Match each term with the correct description.
_____ capsule a. involved in transcriptional
_____ replication regulation
_____ repressor b. able to grow in the presence
_____ tRNA or absence of oxygen
_____ facultative c. maintains selective
anaerobe permeability and cell shape
_____ gram-negative d. provides a mechanism to
_____ gram-positive evade the human immune
_____ aerobic system
_____ cell envelope e. the process of making a
new DNA molecule
f. involved in protein
translation
g. contains a thick layer of
peptidoglycan
h. final electron acceptor is
oxygen
i. has an outer and inner
membrane
Tille: Bailey & Scott’s Diagnostic Microbiology, 14th Edition
Chapter 02: Bacterial Genetics, Metabolism, and Structure

Answers to Case Studies and Review Questions

Review Questions

Multiple Choice

1. b; 2. b; 3. d; 4. a; 5. c

Matching

6. d, e, a, f, b, i, g, h, c

7. d; 8. c

True/False

9. F, T, T

Short Answer

10. Bacteria are prokaryotes; the genetic material is not contained within a nucleus,
allowing replication, transcription, and translation to occur simultaneously.

Copyright © 2017, Elsevier Inc. All Rights Reserved.


CHAPTER 3 Host-Microorganism Interactions 41.e1

Chapter Review
1. An infection acquired from working with an animal 9. All of the following are involved in humoral immunity
reservoir is: except:
a. Acquired from a vehicle a. Cytotoxic T cells
b. Transmitted by a vector b. Complement proteins
c. A zoonotic infection c. Plasma cells
d. An example of indirect transmission d. Glycoproteins
2. Which of the following is considered an indirect mode of 10. Microorganisms that live in or on the human body
transmission? without causing damage include:
a. A cut with a dirty knife a. Colonizers
b. Ingesting contaminated potato salad b. Normal flora
c. Inhaling a droplet containing a bacterium c. Microbiota
d. Drinking water from a contaminated source d. Human microbiome
3. Nonspecific immunity includes all of the following e. All of the above
except: 11. Biofilm formation within a host results in:
a. Inflammation a. Easy clearance of bacterial pathogens
b. Phagocytosis by neutrophils b. Availability of organic nutrients to the host
c. B-cell activation to produce antibodies c. Inability of the immune system to remove the
d. Resident normal microbiota pathogen
4. Humoral immunity: d. Starvation of the microorganisms
a. Is activated for all infectious agents 12. Matching: Match each term with the correct description.
b. Is specific for any organism _____ vector a. injection of antigens or
c. Is specifically targeted to an antigen _____ nosocomial antibodies to provide
d. Provides a broad immune response to any microor- _____ fomite immunity
ganism _____ colonization b. inanimate source of
5. Bacterial endotoxins are: _____ monocytes infection
a. All the same _____ complement c. limited and specific effect
b. Part of the gram-negative cell wall _____ virulence d. long-term health
c. Capable of causing a systemic shock response factor care–associated infection
d. All of the above _____ exotoxin e. characteristic of a disease-
6. A sign is different from a symptom in all of the following _____ immunization causing organism
ways except: f. serum proteins activated
a. It provides measurable data. in the immune system
b. It is believed to be associated with the etiology of the g. circulate in the blood
disease. before activation
c. It is clearly visible. h. insect that carries an
d. It includes the temperature, respiratory rate, and infectious agent
pulse. i. association between
7. A short-lived infection that manifests with a short incu- normal microbiota
bation period and serious illness is considered to be: and host
a. Persistent 13. Short Answer
b. Chronic Compare and contrast the components of the specific
c. Latent and nonspecific immune defenses, including the
d. Acute occurrence and process of inflammation; phago-
8. A microorganism that colonizes the skin but is capable cytic cells; antibody production; cellular response;
of causing infection under the appropriate conditions is and natural physical or chemical properties of the
referred to as: human body.
a. A pathogenic organism
b. An opportunistic pathogen
c. Normal microbiota
d. A nosocomial pathogen
Answers to Case Studies and Review Questions 3-2

T cells PMNS
B cells NK cells
Macrophages Skin (pH, salt, sebum)
Antigen presenting cells Eyes (tears, blinking)
Antibody production Respiratory (mucous, mucociliary escalator, coughing)
Gastrointestinal
Saliva (lysozyme, chewing)
Stomach (pH)
Intestines (peristalsis)
Urine (pH, flushing)

Copyright © 2017, Elsevier Inc. All Rights Reserved.


Tille: Bailey & Scott’s Diagnostic Microbiology, 14th Edition
Chapter 03: Host-Microorganism Interactions

Answers to Case Studies and Review Questions

Case Studies

Case Study 3-1

1. Signs—Measurable
a. CT scan—identifiable mass
b. CBC
c. Microtest results
d. Chemistry, coagulation and hepatitis laboratory results
e. ESR and CRP results
Symptoms—How patient feels
a. Upper abdominal pain
b. Vomiting
c. Headache
d. Fever

2. The patient likely has a chronic condition based on the fact that he has had a general
history of nonspecific symptoms. His emigration and family history along with current
laboratory results are suggestive of an amoebic infection.

Review Questions

Multiple Choice

1. c; 2. a; 3. c; 4. c; 5. d; 6. b; 7. d; 8. b; 9. a; 10. e; 11. c

Matching

h, d, b, i, g, f, e, c, a

Short Answer

Inflammation occurs in both the nonspecific and specific immune response. However,
inflammation begins as an activated process in the nonspecific. Any break in a tissue
provides for the release of chemicals that attract immune cells. PMNs are 80% of the
white blood cells in the circulation and respond to the signals. This results in an
activation cascade of multiple immune factors. This increases blood flow to the site. The
result is warm, red and swelling due to the influx of white cells and fluid.

Specific Nonspecific

Copyright © 2017, Elsevier Inc. All Rights Reserved.


CHAPTER 4 Laboratory Safety 53.e1

Chapter Review
1. Disinfection may be defined as a process that: 9. When disinfecting a contaminated surface, which of
a. Removes all forms of microbial life the following would provide the most efficient decon-
b. Is accomplished by physical means tamination?
c. Removes pathogenic organisms but not spores a. 100% bleach for 10 minutes
d. Is used on living tissue b. 80% bleach for 10 minutes
2. All of the following factors affect the outcome when c. 70% ethyl alcohol for 8 minutes
using a disinfectant except: d. 10% bleach for 5 minutes
a. Using water from a soft water system 10. An engineering control includes:
b. Disinfecting a surface that contains large pores a. Positive air pressure capable of removing all hazard-
c. The type of cloth used to wipe the surface ous contaminants
d. Temperature and pH b. Gloves, laboratory coats, and face shields
3. In a chemical hygiene plan, the MSDS must include: c. Safety caps provided on tubes
a. The facility’s name d. The use of biologic safety cabinets for sample
b. The laboratory director’s credentials processing
c. The substance’s stability and reactivity 11. The laboratory received a stool culture from a patient
d. The purchase date experiencing severe abdominal cramps and bloody
4. Most laboratories use which type of fire extinguisher? diarrhea. The sample should be minimally processed
a. Type A using:
b. Type B a. BSL-1 safety practices
c. Type C b. BSL-2 safety practices
d. Combination ABC c. BSL-3 safety practices
5. When putting out a fire in the laboratory, personnel d. Insufficient information is available to determine
should be sure to: the correct answer.
a. Grab anything that is not replaceable 12. Matching: Match each term with the correct description.
b. Aim directly at the core of the fire _____ Standard a. effective on laboratory
c. Turn off the computer system Precautions benches
d. Activate the fire alarm and contain and extinguish _____ sterilization b. uses a filtering system
the fire, if possible _____ antiseptics c. hazardous chemicals
6. Means of exposure to agents in the laboratory include _____ fume hood d. kills all life forms
all of the following except: _____ incineration e. CDC guidelines
a. Percutaneous inoculation _____ biologic safety f. living tissue
b. Inhalation cabinet g. temperatures exceed
c. Drinking water _____ 10% bleach 870°C
d. Placing objects in the mouth 13. Short Answer
7. Removal of resistant bacterial spores can be accom- A fire has started inside a piece of equipment in the
plished using all of the following methods except: laboratory. Describe the steps the laboratory person-
a. Moist heat at 121°C at 15 psi nel should follow to minimize the damage and cost
b. Incineration from the fire.
c. Moist heat at 132°C for 1 hour
d. Dry heat at 180°C for 3 hours
8. Pasteurization is used to disinfect food products for all
of the following reasons except:
a. To prevent the killing of flavor-adding microorganisms
b. To prevent the destruction of essential vitamins and
minerals in the food
c. To completely sterilize food from contaminating
organisms.
d. To remove food pathogens
Tille: Bailey & Scott’s Diagnostic Microbiology, 14th Edition
Chapter 04: Laboratory Safety

Answers to Case Studies and Review Questions

Review Questions

Multiple Choice

1. c; 2. c; 3. c; 4. d; 5. d; 6. c; 7. a; 8. c; 9. d; 10. d; 11. b

Matching

e, d, f, c, g, b, a

Short Answer

The laboratory personnel should follow the standard procedure as outlined in the safety
protocol, which includes minimizing exposure to hazards to all employees and preventing
injury. The instrument is replaceable.

Copyright © 2017, Elsevier Inc. All Rights Reserved.


CHAPTER 5 Specimen Management 71.e1

Chapter Review
1. All of the following are differential media except: 8. The laboratory received a blood culture specimen and
a. Blood agar placed it directly in the incubator. At 18 hours the cul-
b. Chocolate agar ture indicated positive growth. The technologist com-
c. MacConkey’s agar pleted a Gram stain and plated some of the culture for
d. Eosin methylene blue agar incubation and identification. Identify the discrepancy
2. Selective media: or next step in the process.
a. May be differential a. The technologist must wait at least 18 hours for
b. Allow growth of all organisms growth on the agar plates to identify the organism.
c. Are used for growth of fastidious organisms b. The technologist failed to call the doctor or nurse
d. Contain dyes or antibiotics to suppress the growth of immediately upon completion of the Gram stain
some organisms with the result.
3. A urine specimen was collected at 8 a.m. The nurse c. The technologist completed the process correctly.
labeled the sample and placed it next to the tube station d. The technologist must place the blood culture bottle
to be sent to the laboratory for testing. The specimen was back into the culture system until there is evidence
received in the laboratory for culture at 1 p.m. on the of growth on the agar plates.
same day. The microbiologist should: 9. True or False
a. Set up a routine bacteriology culture _____ Nutritive media supports the growth of all or-
b. Set up a routine bacteriology culture and note the ganisms.
time _____ A requisition should include the patient’s name,
c. Reject the specimen hospital ID, birth date, specimen source, and
d. Call the nurse to explain why the specimen was collection date/time.
rejected and request a new sample _____ EDTA is commonly used to collect blood for
4. Niesseria gonorrhoeae are capnophilic organisms and specimens in the microbiology laboratory.
require: _____ Microbiologic specimens should be set up
a. 5% to 10% CO2, 15% O2 within 2 hours of collection.
b. 5% to 10% H2, 0% O2 _____ Serum samples may be stored frozen for up to
c. 0.3% CO2, 21% O2 1 week.
d. 8% to 10% CO2, 5% to 10% O2 10. Matching: Match each term with the correct description.
5. Specimens may include all of the following except: ____ SPS a. selective
a. Sputum ____ aerobic b. positive CSF
b. Tissue biopsy ____ ambient temperature c. differential
c. Vacutainer needle ____ enrichment d. blood culture
d. Rectal swab ____ Campy systems
6. Enrichment broth is used to: ____ Helicobacter sp. e. requires oxygen
a. Shorten incubation time by providing excess nutrients ____ chocolate agar f. microaerophilic
b. Suppress normal flora to allow pathogens to grow ____ facultative anaerobe g. tryptic soy broth
c. Detect small numbers of anaerobes ____ critical value h. with or without
d. Increase the growth of fastidious organisms ____ blood agar oxygen
7. The laboratory received a tissue biopsy collected in sur- i. room temperature
gery 3 hours earlier from a pancreatic cyst transported in j. fastidious organisms
a gauze pad, covered in saline, and placed in a biohazard 11. Short Answer
bag. The microbiologist should: Explain why direct Gram stains are not typically
a. Set up the tissue specimen after processing to blood performed on throat, nasopharyngeal, or stool
agar (BA), MacConkey, and chocolate agars specimens.
b. Complete a Gram stain and determine whether the
specimen is acceptable
c. Reject the specimen and cancel the order in the labo-
ratory information system
d. Set up the specimen and make a note in the LIS
regarding transport
Tille: Bailey & Scott’s Diagnostic Microbiology, 14th Edition
Chapter 05: Specimen Management

Answers to Case Studies and Review Questions

Review Questions

Multiple Choice

1. b; 2. d; 3. d; 4. a; 5. c; 6. d; 7. d; 8. b

True/False

F, T, F, T, T

Matching

d, e, i, g, a, f, j, h, b, c

Short Answer

Direct Gram stains are not typically performed on throat, nasopharyngeal or stool
specimens because of the large amount of normal microbiota present. Identifying a
pathogen within the Gram stain would be difficult or potentially misleading due to the
numbers of contaminating bacteria.

Copyright © 2017, Elsevier Inc. All Rights Reserved.


CHAPTER 6 Role of Microscopy 88.e1
85.e1

Chapter Review
1. Electron microscopy allows for what magnification of an 7. Matching: Match each of the following terms with the
organism? appropriate description.
a. 1000 _____ Gram stain a. “hot” method of acid-fast
b. 10,000 _____ calcofluor white stain
c. 100,000,000 _____ direct smear b. Gram staining of clinical
d. 1,000,000 _____ carbolfuchsin material
2. A fluorescent dye coupled with specific antibodies is _____ Ziehl-Neelsen c. primary stain in the classic
called: stain acid-fast stain
a. Immunofluorescence _____ safranin d. primary stain of the Gram
b. Fluorochroming _____ Gram’s iodine stain
c. Acridine orange _____ crystal violet e. focuses light path for
d. Electron microscopy _____ Kohler proper resolution
3. The stain that binds to the nucleic acid of organisms but illumination f. counterstain in the Gram
does not discriminate between gram-positive or gram- stain
negative organisms is called: g. stain used to detect fungi
a. Ziehl-Neelsen stain in clinical material
b. Auramine-rhodamine stain h. principal stain used for
c. Gram stain microscopic bacteria
d. Acridine orange stain examination
4. The step in the Gram stain process that distinguishes be- i. mordant that forms the
tween gram-positive and gram-negative organisms is: intracellular complex
a. Fixing of the cells to the slide using heat or methanol 8. Short Answer
b. Decolorization using alcohol or acetone (1) How is contrast achieved in light microscopy?
c. Counterstain of the Gram stain using safranin (2) Calculate the total viewing magnification of an ob-
d. Application of the mordant, Gram’s iodine ject when using the 40 objective lens.
5. Dark-field microscopy is used for the microscopic ex- (3) What two methods of microscopy are most widely
amination of these types of bacteria: used in clinical laboratories?
a. Gram-positive cocci such as Staphylococcus aureus (4) For what reason would the Gram stain of a specimen
b. Yeast such as Candida tropicalis stain gram-variable, and how should that Gram stain
c. Gram-negative bacilli such as Escherichia coli be reported?
d. Spirochetes such as Treponema pallidum (5) An indirect smear from a blood agar plate was Gram
6. True or False stained and reported as “gram-positive cocci in clus-
_____ The ocular lens is the lens closest to the specimen. ters.” However, the Gram stain of the thioglycollate
_____ Resolution is the extent at which detail in a mag- broth demonstrated gram-positive cocci in chains.
nified image is defined. How would the microbiologist resolve this discrep-
_____ Fungi stain gram-negative. ancy, and what should be reported?
_____ Acid-fast bacteria are resistant to decolorization.
_____ A fluorescent dye coupled with specific antibodies
is called fluorochroming.
Tille: Bailey & Scott’s Diagnostic Microbiology, 14th Edition
Chapter 06: Role of Microscopy

Answers to Case Studies and Review Questions

Review Questions

Multiple Choice

1. c; 2. a; 3. d; 4. b; 5. d

True/False

F, T, F, T, F

Matching

h, g, b, c, a, f, i, d, e

Short Answer

1. Use of special dyes and stains.


2. 10 of ocular lens times 40 objective = 400 magnification.
3. Bright field (light) and fluorescent microscopy.
4. Gram-positive organisms that have lost cell wall integrity as a result of antibiotic
treatment, old age, or other reasons will decolorize; report as gram-positive.
5. The smears prepared from each media are the same organism. When preparing a smear
from solid media, it is important to make the smear is thin enough and to look to the outer
edges to find the true morphologic characteristics and arrangement; all organisms tend to
clump together when improperly emulsified. The broth culture allows the microbiologist
to view organisms in their native states, including correct cellular morphologic
characteristics and arrangement; they are not clumped together but are free floating in the
liquid medium.

Copyright © 2017, Elsevier Inc. All Rights Reserved.


CHAPTER 7 Traditional Cultivation and Identification 112.e1

Chapter Review
1. BCYE is an example of which type of media? 9. A clinical isolate was plated to sheep blood, MacConk-
a. Enrichment media ey, and chocolate agars and incubated at 37°C for 24
b. Supportive media hours. The initial plate reading identified medium sized,
c. Combination of selective and enrichment media gamma-hemolytic, shiny gray colonies on sheep blood
d. Differential media agar; small pink colonies on MacConkey agar; and me-
2. CNA media with blood is an example of which type(s) of dium sized, tan, shiny colonies on chocolate agar. The
media? most likely next action for the microbiologist is:
a. Enrichment only a. Identify the two different organisms that are grow-
b. Supportive only ing on the plates as indicated by the size differences
c. Combination of supportive and differential on the agar.
d. Combination of selective and supportive b. Identify the single likely gram-negative lactose
3. Which type of enrichment media is used to isolate Neisseria fermenter as evident by the combined colonial
and Haemophilus organisms? morphology.
a. Hektoen enteric (HE) agar c. Report as normal microbiota without a further bio-
b. Todd-Hewitt broth chemical identification.
c. Regan Lowe agar d. Reinoculate the isolate from the blood and the
d. Chocolate agar MacConkey agar plates to two more sets of primary
4. XLD (xylose-lysine-deoxycholate) agar: media to provide a better preliminary identification
a. Inhibits many gram-negative bacilli that are not enteric of the isolate.
pathogens 10. True or False
b. Inhibits gram-positive organisms _____ Selective media contain one or more agents that
c. Contains a phenol red indicator that detects increased are inhibitory to most organisms except those be-
acidity from carbohydrate ing sought.
d. Causes fermentation _____ Thayer-Martin media support exclusive growth
e. All of the above of the two pathogenic organisms, Neisseria men-
5. The complete clearing of media around bacterial colonies ingitidis and Haemophilus influenza.
on a blood agar plate is referred to as: _____ Most clinically significant gram-negative bacteria
a. Alpha-hemolysis are susceptible to the antibiotic vancomycin.
b. Beta-hemolysis _____ Any zone of bacterial growth inhibition around
c. Gamma-hemolysis a vancomycin-impregnated disk indicates that
d. Delta-hemolysis the bacteria are resistant to the drug.
6. What CO2 concentration is achieved when using a 11. Matching
candle jar? ____ fastidious a. bacteria requiring increased
a. 1% organisms levels of CO2
b. 3% ____ in vivo b. bacteria requiring low
c. 6% ____ oxidase levels of O2
d. 9% negative c. artificial laboratory
7. The following are all bacterial phenotypic characteristics ____ in vitro environment
except: ____ microaerophilic d. catalyzes the release of
a. Microscopic morphology and staining characteristics ____ capnophilic water and O2 from
b. Environmental requirements for growth ____ catalase hydrogen peroxide
c. Nutritional requirements and metabolic capabilities ____ facultatively e. within a living organism
d. The presence of a particular nucleic acid sequence anaerobic f. complex nutritional needs
8. What is the number of bacteria needed to cause turbidity ____ oxidase g. identification of Enterobac-
in broth culture and to be seen with an unaided eye? ____ PYR teriaceae
a. 102 organisms ____ indole h. positive reaction in E. coli
b. 104 organisms i. positive reaction in identifi-
c. 106 organisms cation of Neisseria spp.
d. 108 organisms j. grows in presence or
absence of O2
k. used to differentiate
gram-positive cocci
112.e2 PA RT I I General Principles in Clinical Microbiology

12. Short Answer (5) What is the first and most important step for bac-
(1) What is a biphasic medium, and how is it used? terial identification and characterization?
(2) __________ is inhibitory or lethal for strictly (6) Lack of sufficient water from media is deleterious
anaerobic bacteria. to bacterial growth in what two ways?
(3) What is the main purpose for using a culture (7) Most clinically relevant bacteria can be divided
medium that will meet the bacterial cell growth into four groups based on their Gram stain appear-
requirements? ance; what are these four groups? What does the
(4) What is an example of bacterial pathogens that are term pleomorphic mean?
classified as obligate intracellular parasites, requir-
ing viable host cells for propagation?
Tille: Bailey & Scott’s Diagnostic Microbiology, 14th Edition
Chapter 07: Traditional Cultivation and Identification

Answers to Case Studies and Review Questions

Review Questions

Multiple Choice

1. a; 2. c; 3. d; 4. a; 5. b; 6. b; 7. d; 8. c; 9. b

True/False

T, F, F, F

Matching

f, e, g, c, b, a, d, j, i, k, h

Short Answer

1. A bacterial culture growth media consisting of both a solid (agar) and liquid (broth)
phase; often used in blood cultures.
2. Oxygen.
3. Ensuring that the cell will multiply to sufficient numbers to allow visualization by the
unaided eye to aid in the identification of the bacteria.
4. Chlamydiae and Rickettsia organisms.
5. Selection of a pure colony derived from a single bacterial cell.
6. (1) Less water is available for essential bacterial metabolic pathways; and (2) an
increase in the solute concentration of the media as a result of the loss of water can
osmotically shock the bacterial cell and cause lysis.
7. (1) Gram-positive cocci, (2) gram-negative cocci, (3) gram-positive bacilli, (4) gram-
negative bacilli. The term pleomorphic means exhibiting various shapes, gram-negative
coccobacilli, or gram-variable bacilli.

Copyright © 2017, Elsevier Inc. All Rights Reserved.


CHAPTER 8 Nucleic Acid–Based Analytic Methods for Microbial Identification and Characterization 142.e1

Chapter Review
1. In amplification PCR–based testing, each cycle in the re- 7. Matching: Match each term with the correct description.
action does what to the amount of nucleic acid? _____ homologous a. Reporter molecule
a. Decreases it _____ probe 20-50 bp long
b. Doubles it _____ target b. Chemically linked to
c. Triples it _____ reporter molecule nucleic acid probe
d. Quadruples it _____ gel electrophoresis without antibody
2. What temperature is typically used to achieve DNA de- _____ thermal cycler c. Primers hybridizing to
naturation from a double-stranded molecule to a single- _____ anneal specific nucleic acid
stranded pair of molecules? _____ oligonucleotide (DNA/RNA) target
a. 74°C _____ chemiluminescent d. Nucleic acid from
b. 92°C _____ terminus unknown organism
c. 94°C _____ amplicon e. 3' end of primer
d. 102°C f. PCR amplification
3. In filter hybridization, the membrane is made of what product containing
material? the target nucleic acid
a. Nitrocellulose or nylon fiber of interest
b. Agarose g. Temperature-
c. Ethidium bromide programmable vessel
d. FITC holder
4. Stringency in a hybridization assay is affected by what h. Complementary base
factor(s)? sequence
a. Salt concentration i. Nucleic acid fragments
b. Temperature migrate through
c. Concentration of destabilizing agents agarose gel
d. All of the above j. Forms a complex with
5. In liquid hybridization, what methods are used to elimi- single-stranded DNA
nate background noise? to detect hybridization
a. Enzymatic destruction amplicon
b. Hydroxyapatite or charged magnetic microparticles k. Nucleic acid strand of
c. Chemical destruction known identity
d. All of the above 8. PCR Methodology Matching: Match each term with the
6. True or False correct description.
_____ In real-time PCR assays, the accumulation of am-
plicon is monitored as it is generated. _____ Arbitrary a. Sequential use of two
_____ A disadvantage of SYBR GREEN chemistry is primed PCR primer sets
that it detects only specific amplified products. _____ Quantitative b. Detection of amplicon in
_____ Melting curve analysis in real-time PCR confirms PCR real time
the identity of amplified products and can be per- _____ RT-PCR c. Uses more than one
formed with an assay using hybridization and hy- _____ Real-time PCR primer pair; searches for
drolysis probes. _____ Multiplex PCR different targets
_____ Double-stranded DNA that has cytosine and gua- _____ Nested PCR d. Combines PCR with the
nine bonds requires more heat (energy) to break ability to quantitate the
than DNA containing more adenine and thymi- actual number of targets
dine bases. in the clinical specimen
_____ In real-time PCR, base pair mismatches resulting e. Uses short primers not
from mutations alter the Tm. specifically complementary
_____ The amplification process discriminates real-time to the target DNA
PCR from conventional PCR assays. f. Use of the enzyme reverse
transcriptase to direct
synthesis of DNA from
viral RNA template
142.e2 PA RT I I General Principles in Clinical Microbiology

9. Short Answer (9) Probe-based detection of amplicon serves what two


(1) All molecular methods of testing, regardless of tech- purposes?
nical aspects, involve what? (10) What are the temperature and instrumentation re-
(2) List the four advantages real-time PCR has over con- quirements of isothermal amplification techniques
ventional PCR. compared with conventional PCR?
(3) What are primers and what is their function? (11) Determine the melting temperature for the follow-
(4) What strain typing method has become the “gold ing sequence: GATTCGCAATGGC.
standard” for disease investigation? (12) After the introduction of a single point mutation
(5) What is known as the “blueprint” of the organism? to the sequence in number 11, how would the
(6) What advantage has quantitative PCR added for the melting temperature change for each of the listed
management of some infectious diseases? mutations: T substituted with a G; G substituted
(7) What is the important clinical application of RT- with a C; C substituted with an A?
PCR?
(8) In PCR, name a step that is not required when ana-
lyzing RNA targets.
Tille: Bailey & Scott’s Diagnostic Microbiology, 14th Edition
Chapter 08: Nucleic Acid-Based Analytic Methods for Microbial Identification and
Characterization

Answers to Case Studies and Review Questions

Review Questions

Multiple Choice

1. b; 2. c; 3. a; 4. d; 5. d

True/False

T, F, F, T, T, F

Matching

h, k, d, j, i, g, c, a, b, e, f

PCR Methodology Matching

e, d, f, b, c, a

Short Answer

1. The direct manipulation and analysis of genes (in whole or in part) rather than the
analysis of gene products.
2. Detection of amplicon in real time; No need to open reaction vessel, decreases
contamination; Measures amplicon as it is being made; Less time required (time required
for post PCR detection of amplified product eliminated by the use of fluorescent probes).
3. Primers are short, single-stranded sequences of nucleic acid that are selected to
hybridize (anneal) specifically to a particular nucleic acid target.
4. Pulse field gel electrophoresis.
5. Nucleotide sequence.
6. Presented the ability to monitor the response to drug therapy, detect the ability of drug
resistance, and predict disease progression.
7. The molecular testing of RNA-based viruses; reverse transcriptase directs the synthesis
of DNA from viral RNA template.
8. Denaturation to a single strand of nucleic acid.
9. Allows visualization of PCR product; Provided specificity by ensuring that the
amplicon is the target sequence of interest and not the result of nonspecific amplification.
10. Isothermal means same temperature, therefore does not require a thermal cycler to
complete the reaction. PCR requires a thermal cycler to cycle through temperature
changes.

Copyright © 2017, Elsevier Inc. All Rights Reserved.


Answers to Case Studies and Review Questions 8-2

11. 6 (2) + 7 (4) = 12 + 28 = 40.


12. T substituted with a G, increases by 2; G substituted with a C, no change; C
substituted with an A, decreases by 2.

Copyright © 2017, Elsevier Inc. All Rights Reserved.


CHAPTER 9 Overview of Immunochemical Methods Used for Organism Detection 160.e1

Chapter Review
1. The reasons immunochemical tests are important in b. The lack of fluorescence is a measurement of the anti-
diagnostic testing are: gen-antibody binding and the presence of antibody.
a. The inability to grow organisms on artificial media c. Neutralization of the antigen by the antibody indi-
b. The failure of organisms to survive transport to the cates that antibody is present.
laboratory d. Neutralization of the antigen so that no cytopathic
c. The fastidious nature of organisms and the long incu- effect (CPE) is seen indicates the presence of anti-
bation time for growth in culture body.
d. Antibiotic therapy started before specimen collection 7. What is the acceptable protocol for testing for Lyme dis-
e. All of the above ease, as recommended by the CDC?
2. An example of a precipitation test able to detect antigen a. Screen for the presence of antibody using the ELISA
in a soluble solution is: test; no further testing is required.
a. Particle agglutination test b. Screen for the presence of antibody using the hemag-
b. Coagglutination test glutination test; confirm positive results with the
c. Liposome-enhanced latex agglutination test Western blot test.
d. None of the above c. Screen for the presence of antibody using the ELISA
3. Immunochemical tests are based on what molecular in- test; confirm positive results with the Western blot
teraction? test.
a. Antibody/antigen reaction d. Screen for the presence of antibody using only the
b. Epitope/antigen reaction Western blot test.
c. Myeloma cell/B-cell reaction 8. What substance added to the antigen in the RPR test al-
d. Monoclonal/polyclonal antibody reaction lows for more macroscopically visible flocculation?
4. The laboratory scientist observed an opaque, beta-hemo- a. Latex particles
lytic colony on a sheep blood agar plate of a throat cul- b. Extracellular antigens
ture from a patient. To identify the isolate, the technolo- c. Heparin-magnesium chloride particles
gist retrieved the streptococci typing kit from the d. Charcoal particles
refrigerator. The technologist used the controls included 9. True or False
in the kit and determined the isolate to be positive for _____ One of the important features of the coagglutina-
group A beta-hemolytic streptococci. In reviewing this tion tests is the increased sensitivity.
information: _____ In latex agglutination tests, the minimal reaction
a. The technologist used the improper controls and that can be considered a positive reaction is the
should have used live organisms for the test. grade of 2⫹.
b. The technologist properly performed the test and _____ The term FITC refers to fluorescein isothiocya-
identified the organism. nate, which produces a bright, apple-red fluores-
c. The technologist failed to bring the kit to room tem- cence.
perature, resulting in a false positive reaction. _____ A direct agglutination test detects antibody in the
d. The technologist should have completed additional patient’s serum by binding to latex-coated parti-
biochemical tests for the identification of the or- cles.
ganism. _____ A threefold rise in antibody titer between the
5. A widely used hemagglutination test for detecting anti- acute and convalescent serum samples is consid-
body to T. pallidum is: ered diagnostic of current or recent infection.
a. MHA-TP test _____ Tests developed for use on cerebrospinal fluid
b. FTA-ABS test (CSF) can be used interchangeably with blood se-
c. VDRL test rum specimens.
d. Hemagglutination inhibition assay _____ The anamnestic immune response is the enhanced
6. In the FA test, the antigen is labeled with a compound secondary immune response controlled by mem-
that fluoresces. What determines the presence of anti- ory B lymphocytes.
body? _____ Because the memory cells are not specific, they
a. Measurement of fluorescence is a direct measurement can be stimulated by an antigen to produce anti-
of antigen-antibody binding and the presence of anti- bodies against antigens that are similar but not
body. specific to the original antigen.
_____ The Ouchterlony double immunodiffusion assay
is used to detect antibodies directed against fun-
gal cell components.
160.e2 PA RT I I General Principles in Clinical Microbiology

10. Matching: Match each term with the correct description.


_____ double immunodiffusion a. attached to a fluorescent dye
_____ SPIA b. used to detect B. pertussis, Legionella spp., HSV, and
_____ latex agglutination several viruses
_____ fluorescent antibody test c. FITC directly conjugated to an antibody
_____ antibody conjugation d. epitope
_____ antigenic determinant e. immunoglobulin
_____ IFA f. used to detect exoantigens produced by systemic fungi
_____ DFA g. antibody remains stationary in an agarose matrix
_____ ELISA h. solid-phase immunosorbent assays
_____ antibody i. used to detect C. neoformans in CSF
_____ radial immunodiffusion j. use of a second antibody conjugated to FITC
k. use of the catalytic properties of enzymes to detect
immunologic reactions
11. Matching: Match each term with the correct description.
_____ agglutinins a. process by which phagocytes engulf organisms
_____ titer b. ability to bind more tightly or more specifically
_____ Widal test c. hemagglutination treponemal test for syphilis
_____ chemotaxis d. bind to and block surface receptors on the toxin
_____ acquired immunity e. reciprocal of the highest serum dilution of antibody
_____ humoral immunity f. specific response of the host to an infecting organism
_____ serology g. study of the diagnosis of disease by measuring antibody
_____ avid levels in the serum
_____ cell-mediated response h. antibodies that bind to bacterial antigens, forming visible
_____ antibody-mediated response aggregates
_____ HATTS i. antibodies that bind to the antigens and activate the
_____ opsonizing antibodies complement pathway
_____ neutralizing antibodies j. antibodies circulating in the blood/antibody-mediated
_____ complement-fixing antibodies immune response
k. capable of enhancing phagocytosis
l. typhoid febrile agglutinin test
m. immune response carried out by the T lymphocytes;
includes cytotoxic and helper cells
n. immune response generated by the B lymphocytes;
proteins exhibit immunologic function

12. Short Answer (6) What are two types of disease in which serum anti-
(1) Explain how a hybridoma cell is produced and its body levels may take months to rise or may never
importance in immunochemical testing. rise?
(2) What is the deficiency in the immortal myeloma cell (7) What is the difference between the acute and con-
that allows it to fuse with the antibody-producing valescent blood sera? What antibody class or classes
B cells from the spleen of a mouse? may be identified in each?
(3) Name the three major advantages of using enzymes (8) What antibody class rises during parasitic infections?
as labels in the EIA test. (9) What does the acronym STORCH stand for?
(4) One of the disadvantages of the latex agglutination (10) The presence of IgG antibody or IgM antibody in
test is that some constituents of body fluid (e.g., a newborn’s blood is an indicator of what?
rheumatoid factor) can cause false-positive results. (11) What are two major advantages of using ELISA
How is this problem corrected before testing of a testing in the laboratory?
body fluid? (12) What is the most widely used flocculation test?
(5) What is one major advantage of using immuno- (13) What is an added clinically relevant result of sepa-
fluorescent assays? rating IgM antibody from IgG?
Tille: Bailey & Scott’s Diagnostic Microbiology, 14th Edition
Chapter 09: Overview of Immunochemical Methods Used for Organism Detection

Answers to Case Studies and Review Questions

Review Questions

Multiple Choice

1. e; 2. d; 3. a; 4. c; 5.a, 6. a; 7. c; 8. d

True/False

F, T, F, F, F, F, T, F, T

Matching

f, h, i, b, a, d, j, c, k, e, g

Matching

h ,e, l, a, f, j, g, b, m, n, c ,k, d, i

Short Answer

1. Hybridoma cell is produced by the fusion of a malignant antibody-producing myeloma


cell with an antibody-producing B cell. Offspring of this cell continuously produces
specific monoclonal antibodies.
2. The myeloma cells are deficient in the enzyme, hypoxanthine
phosphoribosyltransferase, and cannot survive in a medium containing hypoxanthine,
aminopterin, and thymidine (HAT medium). The spleen cells of the mouse contain the
enzyme that allows them to thrive in the medium.
3. (1) The enzyme itself is not changed during activity; it can catalyze the reaction of
many substrate molecules, greatly amplifying the reaction and enhancing detection. (2)
Enzyme-conjugated antibodies are stable and can be stored for a relatively long time. (3)
The formation of a colored end product allows direct observation of the reaction or
automated spectrophotometric reading.
4. Pretreating the specimen either by heating or with ethylenediaminetetraacetic (EDTA)
acid.
5. The ability to assess the adequacy of the specimen visually.
6. Legionnaires and hepatitis.
7. The acute serum sample should be collected when disease is first suspected. If this is
the patient’s first exposure to the etiologic agent, then the serum will contain the first
antibody produced in the primary response, IgM. (Note: If the sample is collected too
soon into the disease process, then the serum may only contain low or no antibody titers.)

Copyright © 2017, Elsevier Inc. All Rights Reserved.


Answers to Case Studies and Review Questions 9-2

The convalescent sample should be collected at least 2 weeks after the acute sample is
collected, and the two should both be tested at the same time to avoid variability in the
testing process. If this is a true disease infection, then the IgM antibody should rise to
produce a fourfold rise in antibody titer, which is considered diagnostic of the current
infection. After a second encounter with the etiologic agent, production of antibody will
switch from IgM to IgG, which is called the anamnestic response.
8. IgE.
9. Syphilis, Toxoplasma, rubella, cytomegalovirus, and herpes; these agents are difficult
to culture, and a woman may encounter them in her lifetime, possibly infecting her
newborn child.
10. IgG antibody is able to cross the placenta barrier, and newborns will possess their
mother’s IgG antibodies for the first 2 to 3 months of the infant’s life. IgM antibody
cannot cross the placenta barrier, and the presence of IgM antibody is indicative of
disease.
11. ELISA testing provides the ability to run many samples at one time; the addition of
color changes the substrate and allows the test to be read using instrumentation.
12. Rapid plasma regain (RPR) for syphilis.
13. Removal of rheumatoid factor, which is a patient’s production of IgM antibody
against their own IgG antibody and which can cause nonspecific and interfering test
result.

Copyright © 2017, Elsevier Inc. All Rights Reserved.


CHAPTER 10 Principles of Antimicrobial Action and Resistance 175.e1

Chapter Review
1. Which pair of antimicrobial agents inhibits folic acid 8. Matching: Match each antimicrobial with its mechanism
synthesis? of action. Mechanisms of action choices may be used more
a. Sulfonamides and trimethoprim than once.
b. Chloramphenicol and telithromycin Antimicrobial Class, Example Mechanisms of Action
c. Ceftazidime and avibactam _____ Aminoglycoside, e.g., a. cell wall inhibitor
d. Quinupristin and dalfopristin amikacin b. cell membrane
2. The mechanism of action for tetracycline is inhibition of: _____ Cephalosporin, e.g., inhibitor
a. RNA synthesis ceftazidime c. protein synthesis
b. Cell wall synthesis _____ Polymyxin, e.g., colistin inhibitor
c. Protein synthesis _____ Lipoglycoprotein, e.g., d. DNA synthesis
d. Membrane function dalbavancin inhibitor
3. The inherent ability of a microorganism to resist the ac- _____ Lipoprotein, e.g., e. RNA synthesis
tion of an antimicrobial agent is what type of resistance? daptomycin inhibitor
a. Acquired _____ Macrolide, e.g., f. other metabolic
b. Intrinsic erythromycin processes inhibitor
c. Clinical _____ Fluoroquinolone, e.g.,
d. Biologic levofloxacin
4. The two cations known to influence the activity of ami- _____ Penicillin, e.g.,
noglycosides are: piperacillin
a. Sodium and potassium _____ Ansamycin, e.g., rifampin
b. Calcium and potassium _____ Nitrofurans, e.g.,
c. Calcium and magnesium nitrofurantoin
d. Sodium and magnesium _____ Glycylglycines, e.g.,
5. The most common mechanism of beta-lactam resistance is: tigecycline
a. Alteration of target site _____ Oxazolidinones, e.g.,
b. Decreased permeability tedizolid
c. Increased efflux
d. Enzymatic inactivation
6. Alteration of a drug target site is a method of resistance to:
a. Beta-lactam agents
b. Aminoglycosides
c. Fluoroquinolones
d. All of the above
7. Matching: Match each term with the correct description.
_____ antimicrobial a. biologic resistance
_____ clavulanic acid b. effective killing
_____ beta-lactamase c. physiologic production
_____ porins d. altered target
_____ intrinsic e. therapeutic threshold
resistance f. synthetic
_____ antibiotic g. inhibits growth
_____ PBP2a h. beta-lactamase inhibitor
_____ efflux pump i. enzymatic inactivation
_____ acquired j. decreased uptake
resistance k. diminished accumulation
_____ bacteriostatic l. genetic exchange
_____ clinical resistance
_____ bactericidal
Tille: Bailey & Scott’s Diagnostic Microbiology, 14th Edition
Chapter 10: Principles of Antimicrobial Action & Resistance

Answers to Case Studies and Review Questions

Case Studies

Case Study 10-1

1. Yes. Methicillin and oxacillin belong to the -lactam class of antimicrobial agents.
Mechanisms of resistance, such as an alteration of the drug target site, typically confer
resistance to all drugs in this antimicrobial class.
2. The most likely method is the transfer of the vancomycin resistance gene from
Enterococcus faecalis to Staphylococcus aureus. Resistance genes are easily shared
among bacteria and are the primary means of dissemination of antimicrobial resistance.
Both organisms were isolated from the foot ulcer, providing proximity and opportunity
for genetic transfer.
3. Altered target is the most likely resistance pathway used for both vancomycin-
resistance and methicillin-resistance in this S. aureus isolate.

Review Questions

Multiple Choice

1. a; 2. c; 3. b; 4. c; 5. d; 6. d

Matching—7

f, h, i, j, a, c, d, k, l, g, e, b

Matching—8

o, m, n, m, n, o, p, m, q, r, o, o

Copyright © 2017, Elsevier Inc. All Rights Reserved.


CHAPTER 11 Laboratory Methods and Strategies for Antimicrobial Susceptibility Testing 204.e1

Chapter Review
1. All of the following are considered when standardizing 6. When selecting antimicrobial agents for a susceptibility
antimicrobial susceptibility testing except: testing profile, the laboratorian should consider:
a. Growth medium cation concentration a. Patient population
b. Incubation time b. Identification of the organism
c. Virulence of organism c. Cost of the drug
d. Antimicrobial concentration d. Drug side effects
2. The standard inoculum size for agar dilution susceptibility 7. Which susceptibility profiles require additional testing?
testing is: a. Vancomycin-resistant Staphylococcus aureus
a. 1  104 CFU/mL b. Penicillin-resistant Streptococcus pneumoniae
b. 5  105 CFU/mL c. Gentamicin-resistant Pseudomonas aeruginosa
c. 1  107 CFU/mL d. Erythromycin-resistant viridans streptococci
d. 5  108 CFU/mL 8. Matching: Match each term with the correct description.
3. In a broth dilution susceptibility test, the lowest antimi- _____ indifferent a. substantially less activity
crobial concentration that inhibits visible bacterial growth _____ intermediate b. collected before initial
is known as the: _____ MBC dose
a. Minimal bactericidal concentration _____ peak level c. 99.9% reduction in CFU
b. Minimal inhibitory concentration _____ break point d. no better or worse
c. Susceptibility breakpoint _____ susceptible e. increased activity
d. Resistance breakpoint _____ antagonism f. highest serum
4. Methicillin-resistant Staphylococcus aureus is detected _____ MIC concentration
using agar containing: _____ trough level g. indicator of drug class
a. Penicillin and sodium chloride _____ McFarland resistance
b. Ampicillin and magnesium chloride standard h. separation and definition
c. Nafcillin and magnesium chloride _____ resistant i. absence of resistance
d. Oxacillin and sodium chloride _____ synergy j. may be effective
5. A chromogenic cephalosporin is used to detect resistance to: _____ predictor drug k. inhibits visible growth
a. Macrolides l. 1.5  108 CFU/mL
b. Aminoglycosides m. absence of inhibition zone
c. Beta-lactams
d. Tetracyclines
Tille: Bailey & Scott’s Diagnostic Microbiology, 14th Edition
Chapter 11: Laboratory Methods and Strategies for Antimicrobial Susceptibility
Testing

Answers to Case Studies and Review Questions

Case Studies

Case Study 11-1

1. Uncomplicated infections with Neisseria gonorrhoeae are treated empirically because


the antimicrobial susceptibility pattern is predictable. Most N. gonorrhoeae are resistant
to the original drug of choice, penicillin, but remain susceptible to more potent drugs
such as ceftriaxone and ciprofloxacin. However, recently resistance to ciprofloxacin has
increased and only rarely has resistance to ceftriaxone reported. (Ref: CDC Grand
Rounds: The growing threat of multidrug-resistant gonorrhea. MMWR 62(6):103-106,
2013.)
2. The physician is interested in additional tests because the patient failed empiric therapy
with ciprofloxacin. Reasons for therapy failure include infection with fluoroquinolone-
resistant N. gonorrhoeae or misdiagnosis of the etiologic agent(s) that caused the
patient’s infection. Routine culture verifies the etiologic agent and provides susceptibility
information.
3. The Centers for Disease Control and Prevention (CDC) conducts nationwide
antimicrobial resistance surveillance to determine the incidence of resistance among N.
gonorrhoeae. The 2015 treatment guidelines recommend injectable ceftriaxone plus oral
azithromycin. (Ref: www.cdc.gov/std/gonorrhea/arg/basic.htm) Results from the
surveillance are used to help formulate empiric therapy recommendations and are
available to public health and clinical practitioners.

Case Study 11-2

Questions

1. Would antimicrobial susceptibility tests be done routinely in this scenario?


No, β-hemolytic streptococci, including group B streptococci, remain susceptible to
penicillin. Thus, testing against penicillin provides little, if any, information not already
provided by accurate organism identification.

2. Give a reason susceptibility tests may be needed for this situation.


If the patient cannot tolerate penicillin, alternative agents, such as erythromycin, may be
considered. Because erythromycin resistance among β-hemolytic streptococci has been
well documented, susceptibility testing in this instance is indicated.

3. If the isolate was found to be erythromycin resistant, what additional testing should be
done?

Copyright © 2017, Elsevier Inc. All Rights Reserved.


Answers to Case Studies and Review Questions 11-2

The “D zone test” for inducible clindamycin resistance would need to be performed.

Review Questions

Multiple Choice

1. c; 2. a; 3. b; 4. d; 5. c; 6. b; 7. a

Matching

d, j, c, f, h, i, a, k, b, l, m, e, g

Copyright © 2017, Elsevier Inc. All Rights Reserved.


CHAPTER 13 Staphylococcus, Micrococcus, and Similar Organisms 261.e1

Chapter Review
1. A clinical isolates test demonstrated the following results: 7. Matching
catalase-positive, gram-positive cocci, nonhemolytic on _____ antiphagocytic a. staphylokinase
blood agar plate, coagulase-negative, novobiocin resistant, _____ S. saprophyticus b. toxic shock syndrome
PYR negative, and glucosidase-positive. The organism is _____ Panton Valentine c. endocarditis
most likely: leukocidin d. alpha toxin
a. S. cohnii _____ S. epidermidis e. polysaccharide capsule
b. S. saprophyticus _____ sphingomyelinase f. urinary tract infection
c. S. xylosus _____ pyrogenic g. scalded skin syndrome
d. S. epidermidis exotoxin C h. PYR positive
2. Which of the following organisms is coagulase positive? _____ S. simulans i. lyses white blood cells
a. S. saprophyticus _____ beta hemolysin j. associated with medical
b. S. haemolyticus _____ exfoliative devices
c. S. hominis _____ fibrinolysin k. beta toxin
d. S. pseudintermedius _____ S. haemolyticus l. novobiocin resistant
3. The D-zone susceptibility test is used to test inducible _____ S. sciuri
resistance on S. aureus strains demonstrating an initial
antibiotic susceptibility profile of: 8. Short Answer: Examine the following isolated antibiotic
a. Erythromycin sensitive, clindamycin sensitive sensitivity profiles for S. aureus. Identify the methicillin-
b. Erythromycin resistant, clindamycin sensitive resistant organism based on the pattern of resistance and
c. Erythromycin resistant, clindamycin resistant sensitivities.
d. Erythromycin sensitive, clindamycin resistant 1. Isolate 1
4. All of the following media used for the cultivation of Antibiotic Automated Result Final Result
Micrococcaceae are selective except: Beta-lactamase POS
a. 5% sheep blood agar Cefoxitin screen NEG  NEG
b. Phenylethyl alcohol agar Clindamycin 0.25 S
c. Mannitol salt agar Erythromycin 0.25 S
d. Colistin nalidixic acid agar Inducible NEG  NEG
5. A blood culture isolate grew as a large, white, nonhemo- clindamycin
lytic colony on 5% sheep blood agar and was coagulase Oxacillin 0.5 S
negative (slide method). The microbiologist should: 2. Isolate 2
a. Report as coagulase-negative staphylococci, probable
contaminate Antibiotic Automated Result Final Result
b. Follow up with an oxidase and bacitracin test before Beta-lactamase POS
reporting Cefoxitin screen POS  POS
c. Complete a tube coagulase test Clindamycin 0.25 S
d. Repeat the slide coagulase test with new controls Erythromycin 8 R
6. True or False Inducible NEG  NEG
_____ Micrococcaceae commonly appear as tetrads, clindamycin
chains, and singlets on Gram stains. Oxacillin 4 R
_____ All S. aureus clinical isolates are coagulase-positive 3. Isolate 3
in both slide and tube methods.
Antibiotic Automated Result Final Result
_____ S. lugdunensis is an important isolate found in
Beta-lactamase POS
wounds as a result of a dog bite.
Cefoxitin screen NEG  NEG
_____ Bacitracin and oxidase tests are sufficient to dif-
Clindamycin 0.25 S
ferentiate Micrococcus spp. from other coagulase-
Erythromycin 0.25 S
negative gram-positive cocci.
Inducible NEG  NEG
_____ Micrococcaceae are predominately facultative an-
clindamycin
aerobes.
Oxacillin 0.5 S
Tille: Bailey & Scott’s Diagnostic Microbiology, 14th Edition
Chapter 13: Staphylococcus, Micrococcus, and Similar Organisms

Answers to Case Studies and Review Questions

Case Studies

Case Study 13-1

1. PYR, β-galactosidase, and VP will differentiate among the tube coagulase-positive


staphylococci. Staphylococcus intermedius is VP-negative and PYR-positive and is a
pathogen in dogs. This isolate was S. intermedius, which was probably acquired by the
mother while working in the veterinarian’s office and then transmitted to her son.
2. An ornithine decarboxylase test should be performed on PYR-positive staphylococci
from significant normally sterile sites to identify Staphylococcus lugdunensis, which is
among the few staphylococci that have a positive reaction for this test. S. lugdunensis can
be problematic to identify because it can be slide coagulase-positive but is always tube
coagulase-negative. Fortunately, it does not have the typical β-hemolysis of S. aureus.
However, S. aureus has been identified that no longer expresses the β-hemolytic
phenotype, resulting in misidentification as S. lugdunensis. The identification of S.
lugdunensis is important, because this organism is highly associated with infectious
endocarditis, a complication of bacteremia. Treatment must be long and aggressive.
Because more than half of S. lugdunensis are β-lactamase–negative, the laboratory should
include this result in the report, so that penicillin can be considered for therapy.
3. The only known mechanism of resistance is mediated by the mecA gene in
staphylococci, which produces penicillin-binding protein 2a (PBP2a). This protein does
not bind well to either penicillin or the penicillinase-resistant penicillins (i.e., oxacillin,
dycill [dicloxacillin], nafcillin). Thus this protein is available to the cell to complete the
formation of the cell wall. Direct detection of the gene or its protein product is an
excellent method to detect resistance in coagulase-negative staphylococci.
4. Surprisingly, the agent is cefoxitin, a drug that would not generally be considered to
treat staphylococcal infections. The disk test is recommended by the Clinical and
Laboratory Standards Institute (CLSI) as the best predictor of both susceptibility and
resistance, having a high sensitivity and specificity.1
1
Clinical and Laboratory Standards Institute: Performance standards for antimicrobial
susceptibility testing; M100-S23, Wayne, PA, 2013, CLSI.

Case Study 13-2

1. Gram-positive cocci in clusters, many white blood cells (WBCs).


2. Blood cultures, as well as a culture and sensitivity of the joint fluid, would be
recommended to ensure that the patient was not developing a bacteremia. Blood cultures
in this case were negative at 48 hours in both the aerobic and anaerobic cultures. Joint
fluid demonstrated the growth of a coagulase-negative staphylococcus.

Copyright © 2017, Elsevier Inc. All Rights Reserved.


Answers to Case Studies and Review Questions 13-2

3. The patient has developed a septic arthritis as a result of an infection with a coagulase-
negative staphylococcus.
4. Additional testing would include a biochemical panel and antibiotic sensitivity profile
to confirm the identity of the organism and proper treatment of the patient. The joint was
replaced with an antibiotic impregnated prosthesis. A peripherally inserted central
catheter (PICC) line was inserted, and she was treated for 6 weeks with intravenous (IV)
antibiotics. Eventually, the patient received a new permanent joint replacement.

Review Questions

Multiple Choice

1. b; 2. d; 3. b; 4. a; 5. c

True/False

F, F, F, T, T

Matching

e, f, i, j, k, b, h, d, g, a, c, l

Short Answer

1. Methicillin-sensitive S. aureus, as evidenced by oxacillin sensitivity and negative


cefoxitin screen; β-lactamase–positive staphylococci are considered resistant to all
penicillins.
2. Methicillin-resistant S. aureus, as evidenced by oxacillin resistance and positive
cefoxitin screen; all β-lactamase–positive, oxacillin-resistant staphylococci are
considered resistant to cephems, β-lactam and β-lactamase inhibitor combinations,
carbapenems, and penicillins.
3. Methicillin-sensitive S. aureus, as evidenced by oxacillin sensitivity and negative
cefoxitin screen; β-lactamase–positive staphylococci are considered resistant to all
penicillins.

Copyright © 2017, Elsevier Inc. All Rights Reserved.


CHAPTER 14 Streptococcus, Enterococcus, and Similar Organisms 281.e1

Chapter Review
1. Necrotizing fasciitis is a serious infection associated with: 9. What bacteria cause neonatal sepsis in 1- to 5-day-old
a. S. agalactiae infants?
b. S. mitis a. S. pyogenes
c. S. pyogenes b. S. agalactiae
d. S. epidermidis c. Enterococcus spp.
2. Rheumatic fever is a poststreptococcal sequelae typically d. Viridans streptococci
associated with: 10. Sputum specimen inoculated on 5% sheep blood agar
a. Postpartum infections associated with group B reveals alpha-hemolytic grayish colonies that are very
streptococci mucoid. What test should be set up next?
b. Skin infections and pyodermas associated with group a. PYR
A streptococci b. Optochin
c. Pharyngitis associated with group A streptococci c. Bacitracin
d. Pneumonia associated with S. pneumoniae d. CAMP
3. Which of the following organisms are PYR positive? 11. What is the antibiotic of choice for beta-hemolytic
a. Group A Streptococcus Streptococcus?
b. Group B Streptococcus a. Penicillin
c. S. urinalis b. Tetracycline
d. Enterococcus spp. c. Bacitracin
e. a and c only d. Vancomycin
f. a, c, and d 12. True or False
4. When streaking a throat culture on 5% sheep blood agar, _____ Glomerulonephritis is a poststreptococcal sequel-
stabbing the agar plate provides enhanced detection for: ae that always follows S. pyogenes pharyngitis.
a. Bile solubility _____ Aerococcus spp. may be isolated from blood cultures
b. Bile esculin hydrolysis only.
c. DNase activity _____ Nutritionally variant streptococci are typically
d. Streptolysin O cultivated on blood agar and may or may not
e. Streptolysin S grow in the presence of NaCl.
5. Which organism produces the CAMP factor enhancing _____ Gemella are gram-positive cocci that may appear
beta-hemolysis in the presence of the S. aureus beta-lysin? as pairs or clusters but predominately appear as
a. Group A streptococci chains upon Gram staining.
b. Group B streptococci _____ Colony morphology of Pediococcus resembles
c. Group C streptococci viridans streptococci and is alpha-hemolytic or
d. Group D streptococci gamma-hemolytic.
6. Optochin sensitivity is used to differentiate: 13. Matching: Match each term with the correct description.
a. S. pneumoniae from other alpha-hemolytic streptococci _____ M protein a. toxic shock syndrome
b. S. pyogenes from S. pneumoniae _____ secretory IgA b. dissemination of
c. S. agalactiae from S. pyogenes protease blood infections
d. Enterococci from non–group D enterococci _____ impetigo c. anti-DNase positive,
7. A 24-hour growth on a urine culture from a 20-year-old _____ streptolysin S antistreptolysin
woman demonstrates grayish white beta-hemolytic _____ streptolysin O O negative
colonies, which are catalase negative. What would be the _____ streptokinase d. S. pneumoniae
next step for the microbiologist? _____ scarlet fever infiltrate
a. Report the organism as S. pyogenes. _____ glomerulonephritis e. degrades mucosal
b. Report the organism as S. aureus. _____ streptococcal antibodies
c. Perform a PYR test. pyrogenic exotoxins f. associated with otitis
d. Perform a CAMP test and hippurate hydrolysis. _____ pneumolysin media
8. Which organism is able to hydrolyze esculin and is a _____ Viridans g. cross-reactive
serious nosocomial pathogen? streptococci antibodies to heart
a. S. bovis _____ Alloiococcus spp. tissue
b. S. mitis h. oxygen labile
c. E. faecalis i. low virulence
d. S. pneumoniae associated with
dental caries
j. oxygen stable
k. pharyngitis
l. skin blisters
Tille: Bailey & Scott’s Diagnostic Microbiology, 14th Edition
Chapter 14: Streptococcus, Enterococcus, and Similar Organisms

Answers to Case Studies and Review Questions

Case Studies

Case Study 14-1

1. The organism is an enterococci that has become vancomycin-dependent. To isolate it, a


vancomycin disk is placed on the blood plate in the area of the inoculum from the blood
culture bottle.
2. A brain-heart infusion agar plate with 6 µg/mL can be used. Inoculate 1 to 10 mL of a
0.5 suspension of the isolate equal to a 0.5 McFarland turbidity standard, and incubate for
24 hours. Greater than one colony of growth is considered presumptive for resistance.
Alternatively, an enterococcosel agar supplemented with 6 g/mL of vancomycin can be
used as a screening plate for the detection of vancomycin-resistant enterococci (VRE).
Any amount of growth is considered resistant.
3. Other than enterococci, two genera are recognized to be intrinsically resistant to
vancomycin. Leuconostoc are gram-positive cocci in chains; they are PYR-variable and
are identified by their characteristic production of gas in de Man, Rogosa, and Sharpe
(MRS) broth (Procedure 12-28). Pediococci are gram-positive cocci in clusters and
tetrads; they are PYR-negative and do not produce gas in MRS broth. Although Weissella
confusa are also vancomycin-resistant and produce gas in MRS broth, they are generally
rod shaped. The best way to demonstrate their rod shape is to prepare the Gram stain
from growth in broth. They are also arginine decarboxylase–positive, unlike Leuconostoc
or Pediococcus.

Case Study 14-2

1. The man is more than 70 years of age and has diabetes, which provides for a host of
problems associated with metabolism and immune depression. He is at risk for chronic
pulmonary problems as a result of exposure to secondhand smoke for an extended period
(35 years).
2. The patient appears to have mild dehydration, as evidenced by the slight elevation in
sodium, as well as the increased blood urea nitrogen (BUN)-to-creatinine ratio. The WBC
result, in combination with the arterial blood gases, clearly points toward a respiratory
ailment caused by an extended exposure to secondhand smoke and the potential
development of bacterial pneumoniae. The discolored sputum should be collected and
cultured along with potential x-ray studies for infiltration in the patient’s lungs.
3. The Gram stain is insignificant, indicating normal microbiota. It is possible that the
patient is unable to expectorate a sufficient sample from the lungs to isolate potential
pathogens. The patient’s history of coughing up dark-colored sputum on a daily basis for
several years could be preventing the patient from pulling sputum from deep in the lower
lungs, masking the infection. Blood cultures should be indicated to search for a more

Copyright © 2017, Elsevier Inc. All Rights Reserved.


Answers to Case Studies and Review Questions 14-2

systemic condition. Blood cultures were obtained, and after 48 hours of incubation they
revealed gram-positive, lancet-shaped cocci, indicative of Streptococcus pneumoniae.
The patient was diagnosed with pneumococcal pneumoniae and placed on antibiotic
treatment.

Review Questions

Multiple Choice

1. c; 2. b; 3. f; 4. d; 5. b; 6. a; 7. d; 8. c; 9. b; 10. b; 11. a

True/False

F, F, T, T, T

Matching

g, e, l, j, h, b, k, c, a, d, i, f

Copyright © 2017, Elsevier Inc. All Rights Reserved.


CHAPTER 15 Bacillus and Similar Organisms 292.e1

Chapter Review
1. The virulence factor associated with B. cereus is: 6. True or False
a. Edema toxin _____ Bacillus spp. are the only organisms to produce
b. Lethal toxin spores in the presence of oxygen.
c. Protective antigen _____ Bacillus spp. are rarely found to be laboratory con-
d. Enterotoxin taminants.
2. Inhalation or pulmonary anthrax is also known as: _____ B. cereus is resistant to penicillin.
a. Black eschar _____ Rapid tests for the presumptive identification of
b. Woolsorters’ disease B. anthracis are not available.
c. Legionnaires’ disease _____ Vegetative cell size is used to differentiate species
d. Plague with the genera Bacillus and Paenibacillus.
3. A large, aerobic, gram-positive, spore-forming rod is iso- _____ Bacillus spp. grow on sheep blood agar and
lated from a blood culture. It can be further confirmed as phenylethyl alcohol agar.
B. anthracis if it is: 7. Matching: Match each term with the correct description.
a. Hemolytic and motile _____ eschar a. B. anthracis vaccine
b. Hemolytic and nonmotile _____ woolsorters’ disease b. B. anthracis isolation
c. Nonhemolytic and motile _____ lethal toxin media
d. Nonhemolytic and nonmotile _____ malachite green c. spore stain
4. A large, aerobic, beta-hemolytic, gram-positive rod is _____ PLET d. B. anthracis virulence
isolated from an eye culture. Subsequent testing reveals it _____ Medusa head factor
is motile and produces a wide zone on egg yolk agar. The _____ endophthalmitis e. pulmonary anthrax
most likely identification of this organism is: _____ pyogenic factor f. B. anthracis
a. Bacillus anthracis _____ Biothrax prophylaxis
b. Escherichia coli _____ ciprofloxacin g. B. anthracis colonies
c. Bacillus cereus h. cutaneous anthrax
d. Clostridium perfringens i. B. cereus infection
5. The most appropriate therapy for inhalation anthrax is: j. B. cereus virulence
a. Ciprofloxacin factor
b. Tetracycline
c. Vancomycin
d. Erythromycin
Tille: Bailey & Scott’s Diagnostic Microbiology, 14th Edition
Chapter 15: Bacillus and Similar Organisms

Answers to Case Studies and Review Questions

Case Studies

Case Study 15-1

1. The β-hemolytic spore-formers are easily separated from the other species of Bacillus
and related genera by their colonial morphology. Bacillus mycoides displays a rhizoid
colony that resembles a fungus. Bacillus thuringiensis is not a typical human pathogen; it
is pathogenic to insects. Bacillus cereus should also be considered in the differential,
which is known to have virulence factors.
2. B. cereus has a β-lactamase, which is more reliably detected by the observation of
growth up to the edge of a 10-unit penicillin disk. β-Lactamase testing is less reliable.
The organism is also motile, which separates it from Bacillus anthracis, the other cause
of serious human infections with Bacillus spp. However, B. anthracis is not hemolytic. A
positive lecithinase test (Figure 15-6) unequivocally confirms the identification, although
this test is rarely needed if the colony is typical.
3. This organism could potentially be B. anthracis. Such an isolate could provide proof
that a bioterrorist event is occurring. All plates and tubes should be immediately placed in
a biological safety cabinet. Then the laboratory supervisor should be notified to contact
the physician, the local epidemiologist, and the local public health department. They will
provide instructions as to what action should be taken and how to handle the cultures.
They will want the Gram stain to be carefully examined. The bacillary vegetative cells
should be greater than 1 µm wide, and the spores should not be wider than the vegetative
cells. B. anthracis is nonmotile, whereas most other bacilli are motile. Old cultures of
saprophytic Bacillus spp. tend to be nonmotile. The preparation of a fresh culture plate
that is microscopically observed in a wet mount after a few hours at 35°C may be all that
is needed to rule out B. anthracis. Performing this test presents no danger; the spores will
not form in such a short time.

Review Questions

Multiple Choice

1. d; 2. b; 3. d; 4. c; 5. a

True/False

T, F, T, F, T, T

Matching

Copyright © 2017, Elsevier Inc. All Rights Reserved.


Answers to Case Studies and Review Questions 15-2

h, e, d, c, b, g, i, j, a, f

Copyright © 2017, Elsevier Inc. All Rights Reserved.


CHAPTER 16 Listeria, Corynebacterium, and Similar Organisms 307.e1

Chapter Review
1. Which two tests can be used to differentiate C. diphtheriae 6. True or False
from other corynebacteria? _____ Corynebacteria are non–spore-forming, catalase-
a. Halo on Tinsdale and positive nitrate positive, gram-positive rods.
b. Halo on Tinsdale and positive urea _____ Corynebacterium and Listeria spp. grow on Mac-
c. Black on Tinsdale and positive esculin Conkey agar.
d. Black on Tinsdale and positive nitrate _____ L. monocytogenes can be presumptively identified
2. All of the following can be used to detect toxin from by observation of motility by direct wet mount.
C. diphtheriae except: 7. Matching: Match each term with the correct description.
a. Guinea pig lethality test _____ lipophilic a. a selective and
b. Elek test _____ cold enrichment differential media for
c. Modified Tinsdale _____ Tdap C. diphtheriae
d. PCR _____ Elek b. used to detect
3. Which organism exhibits end-over-end tumbling motil- _____ modified Tinsdale C. diphtheriae toxin
ity when incubated in nutrient broth at room tempera- _____ Corynebacterium c. Chinese letter
ture for 1 to 2 hours? Gram stain morphology
a. C. diphtheriae _____ Listeria spp. d. grown at 4°C for
b. C. jeikeium several weeks
c. Arthrobacter sp. e. diphtheria, pertussis,
d. L. monocytogenes and tetanus vaccine
4. Pregnant women and immunocompromised patients f. lipid loving
should avoid eating which of the following foods to pre- g. umbrella-shaped
vent Listeria infection? motility
a. Feta cheese
b. Peanuts
c. Pickles
d. Ice cream
5. This strict aerobic organism is commonly associated with
urinary tract infection and is catalase-positive, nitrate-
negative, and rapidly urease positive.
a. C. jeikeium
b. C. urealyticum
c. R. equi
d. D. hominis
Tille: Bailey & Scott’s Diagnostic Microbiology, 14th Edition
Chapter 16: Listeria, Corynebacterium, and Similar Organisms

Answers to Case Studies and Review Questions

Case Studies

Case Study 16-1

1. The technologist performed a wet mount motility test, which was positive for the
characteristic “tumbling motility” of Listeria. In the confirmatory tube motility test at
20°C to 25°C, an umbrella-shaped growth of the organism was formed at the top of the
tube (Figure 16-7). A bile-esculin test was also performed, which was positive. Both tests
indicated that the isolate was Listeria monocytogenes. Laboratory identification protocols
should require checking for hemolysis on all gram-positive rods and include both motility
and bile-esculin tests for isolates that are hemolytic.
2. Susceptibility testing is not routinely recommended because these organisms are
predictably susceptible to ampicillin. Treatment is generally with ampicillin and an
aminoglycoside.
3. Corynebacteria that are pathogens in the urinary tract are urease-positive. They include
Corynebacterium urealyticum, Corynebacterium glucuronolyticum, and
Corynebacterium riegelii. Only C. glucuronolyticum is Christie, Atkins, Munch-Petersen
(CAMP) positive. C. urealyticum is lipophilic. All of these species grow slowly and
might not be detected if the urine cultures are incubated for less than 24 hours.

Review Questions

Multiple Choice

1. b; 2. c; 3. d; 4. a; 5. b

True/False

T, F, T

Matching

f, d, e, b, a, c, g

Copyright © 2017, Elsevier Inc. All Rights Reserved.


CHAPTER 17 Erysipelothirix, Lactobacillus, and Similar Organisms 315.e1

Chapter Review
1. A presumptive identification of G. vaginalis is sufficient 5. Matching: Match each term with the correct description.
for genital isolates, based on which diagnostic test? _____ CNA a. sodium polyanethol
a. Beta-hemolytic HBT agar _____ SPS sulfonate
b. Negative oxidase _____ HBT b. Haemophilus-Neisseria
c. Negative catalase _____ HNID identification panel
d. All of the above _____ clue cells c. associated with bacterial
2. Which of the following is the only catalase-negative, _____ Gardnerella vaginosis
gram-positive, non–spore-forming rod that produces _____ Erysipelothrix d. resemble beta-hemolytic
H2S on TSI? _____ Arcanobacterium streptococci
a. Gardnerella spp. e. Columbia colistin-
b. Erysipelothrix spp. nalidixic acid
c. Lactobacillus spp. f. large, squamous
d. Arcanobacterium spp. epithelial cells
3. Which of the following organisms has multiple colonial g. large and smooth
morphologies and may have alpha-hemolytic colonies colony types
resembling streptococci? h. human blood bilayer
a. Lactobacillus spp. Tween
b. Gardnerella spp.
c. Arcanobacterium spp.
d. Bifidobacterium spp.
4. True or False
_____ E. rhusiopathiae stains as both short rods and long
filaments.
_____ A wet mount of Lactobacillus spp. demonstrates
characteristic clue cells.
_____ Individuals exposed to Lactobacillus spp. should
follow prophylaxis guidelines.
_____ In vitro testing is necessary to guide therapy for
Erysipelothrix infection.
Tille: Bailey & Scott’s Diagnostic Microbiology, 14th Edition
Chapter 17: Erysipelothrix, Lactobacillus, and Similar Organisms

Answers to Case Studies and Review Questions

Case Studies

Case Study 17-1

1. Erysipelothrix rhusiopathiae is the only catalase-negative rod to produce hydrogen


sulfide, visible on triple sugar iron (TSI) agar, Kligler iron agar, or sulfide indole motility
(SIM) agar. This test should be part of the laboratory protocol for the identification of
catalase-negative, gram-positive rods from wounds and blood cultures. Because the
organism grows slowly, the test may take up to 3 days for a positive result. E.
rhusiopathiae is a known, but rare, cause of endocarditis, with a 38% fatality rate. The
disease is indolent (causes relatively no pain or evident pathology), and the patient was
probably infected for a long time.
2. E. rhusiopathiae is intrinsically resistant to vancomycin, which might be considered
for therapy; vancomycin is often used as empiric therapy for infections with gram-
positive bacteria. The organism is susceptible to penicillin. Lactobacillus is the only other
catalase-negative gram-positive rod that is resistant to vancomycin. These two genera are
often confused.
3. Erysipelothrix is an animal pathogen. The patient likely became infected from
exposure to the marine environment, probably through an open wound originating from a
fishhook.

Review Questions

Multiple Choice

1. d; 2. b; 3. a

True/False

T, F, F, F

Matching

e, a, h, b, f, c, g, d

Copyright © 2017, Elsevier Inc. All Rights Reserved.


CHAPTER 18 Nocardia, Streptomyces, Rhodococcus, and Similar Organisms 326.e1

Chapter Review
1. Which organisms may stain partially acid-fast when 5. Which actinomycete is partially acid-fast, has extensive
stained with a modified acid-fast stain? aerial hyphae, and is lysozyme resistant?
a. Gordonia sp. a. Nocardia sp.
b. Rhodococcus sp. b. Rhodococcus sp.
c. Nocardia sp. c. Gordonia sp.
d. All of the above d. Tsukamurella sp.
2. Which organism is most commonly associated with 6. Matching: Match each term with the correct description.
human disease, particularly in immunocompromised _____ mycetoma a. no mycolic acid in cell
patients, such as those infected with HIV? _____ partially acid-fast wall
a. Nocardia asteroides _____ substrate hyphae b. mycetoma caused by
b. Rhodococcus equi _____ aerial hyphae actinomycetes
c. Gordonia sp. _____ actinomycetoma c. fungal isolation agar
d. Tsukamurella sp. _____ non–acid-fast d. branched filaments in
3. Infection caused by non–acid-fast aerobic actinomycetes _____ SABS the air
is usually chronic, granulomatous lesions of the skin _____ Ziehl-Neelsen e. chronic subcutaneous
referred to as: infection
a. Mycelium f. branched filaments along
b. Necrosis agar surface
c. Impetigo g. acid-fast stain
d. Mycetoma h. mild acid decolorization
4. If the Gram stain morphology or colonial morphology is
suggestive of a possible actinomycete, which test should
immediately follow?
a. Ziehl-Neelsen (Acid-fast)
b. Modified acid-fast
c. Urea hydrolysis
d. Nitrate reduction
Tille: Bailey & Scott’s Diagnostic Microbiology, 14th Edition
Chapter 18: Nocardia, Streptomyces, Rhodococcus, Oerskovia, and Similar
Organisms

Answers to Case Studies and Review Questions

Case Studies

Case Study 18-1

1. A penicillin disk will separate most of the contaminating coryneforms, which will be
susceptible to penicillin, from the more pathogenic actinomycetes. If the isolate is
resistant to penicillin, then a positive partial acid-fast stain will separate Nocardia spp.
from saprophytic actinomycetes and coryneforms. This test is difficult to perform and
should always have controls on the same slide as the unknown culture. Young colonies
may give a false-negative result. Definitive identification takes much longer for the
demonstration of microscopic aerial hyphae on tap water agar (18 g/L of agar, in tap
water) and a positive lysozyme test. It is important that the microbiologist not wait for
these latter tests to turn positive to alert the physician of the possibility of the presence of
this serious infection.
2. Nocardia farcinica is resistant to all three classes of agents, making it extremely
difficult to treat. Treatment with antimicrobial agents is usually for 6 to 12 months.
Generally, sulfamethoxazole-trimethoprim is one of the drugs used. In this case, the
woman probably had multiple brain abscesses that spilled into the cerebrospinal fluid
(CSF). The reason the chocolate agar was negative probably had to do with the low
numbers of cells in the specimen and the fact that enriched media can be inhibitory to
Nocardia.
3. Rhodococcus equi is suspected when mucoid, pink colonies of gram-positive rods are
isolated. The characteristic colony plus the ability to be CAMP test–positive and urease-
positive identifies this species. R. equi is a serious pathogen in patients who are
immunocompromised.

Review Questions

Multiple Choice

1. d; 2. b; 3. d; 4. a; 5. a 6. a

Matching

e, h, f, d, b, a, c, g

Copyright © 2017, Elsevier Inc. All Rights Reserved.


CHAPTER 19 Enterobacteriaceae 354.e1

Chapter Review
1. All of the following organisms are considered normal 8. A patient presents with diarrhea and abdominal cramp-
intestinal microbiota except: ing. The organism isolated from the stool culture is
a. C. freundii identified as S. dysenteriae (group A). The TSI reaction
b. Y. enterocolitica would have indicated:
c. E. coli a. K/K
d. E. aerogenes b. K/NC H2S 
2. Enterobacteriaceae are typically gram negative and: c. A/A
a. Non–glucose fermenters d. K/A
b. Capable of reducing nitrates to nitrites 9. Which organism is commonly considered an extrain-
c. Catalase negative testinal pathogen?
d. Oxidase positive a. E. coli
3. A patient presents with a urinary tract infection. After b. E. tarda
24 hours of incubation, the urine culture grows a non– c. P. shigelloides
lactose fermenter on MacConkey agar, colorless colo- d. H. alvei
nies on HE, indole-positive organism. The isolate is e. K. oxytoca
most likely: 10. Matching: Match each term with the correct description.
a. Citrobacter spp. _____ Salmonella a. LPS
b. Escherichia spp. serotype Typhi b. capsular
c. Klebsiella spp. _____ MHT c. prodigiosin
d. Proteus spp. _____ STEC d. deoxycholate
4. Incubation of which organism at 25°C produces a _____ Vi antigen inhibition
characteristic yellow pigment? _____ CIN e. MDRTF
a. C. sakazakii _____ AmpC f. extended-spectrum
b. P. shigelloides _____ Y. pestis cephalosporin
c. E. aerogenes _____ O antigen g. O157:H7
d. H. alvei _____ K. pneumoniae h. carbapenemase
5. The most common cause of hemolytic uremic syn- _____ GN i. HUS
drome is: _____ SS j. plague
a. EPEC _____ H antigen k. bile salts and brilliant
b. EAEC _____ S. marcescens green
c. O157:NM _____ MAC-SOR l. pyogenic liver abscess
d. O157:H7 _____ XLD m. flagella
6. Which E. coli produces a heat-labile (LT) enterotoxin n. H2S production
and a heat-stable enterotoxin? o. mannitol and neutral
a. UPEC red
b. ETEC
c. MNEC
d. EAEC
7. A patient presents to the physician with pain and frequency
of urination. The urine culture reveals a non–lactose fer-
menting, gram-negative rod with characteristic swarming
on blood agar. The biochemical test that would specifically
distinguish this organism from other Enterobacteriaceae is:
a. Lactose fermentation
b. Oxidase
c. Phenylalanine deaminase and H2S
d. Triple sugar iron agar
354.e2 PA RT I I I Bacteriology

11. Short Answer: Interpret the following susceptibility patterns Interpretation:


for two E. coli isolates. _____________________________________________
RESULTS ______________________
Antibiotic Organism 1 Organism 2 _____________________________________________
Amikacin 2 S 16 S ______________________
Ampicillin 2 S 32 R _____________________________________________
Ampicillin/ 2 S 32 R ______________________
Sulbactam
Cefazolin 4 S 64 R _____________________________________________
Cefepime 1 S 1 S? ______________________
Cefoxitin 4 S 4 S _____________________________________________
Ceftazidime 1 S 2 S? ______________________
Ceftriaxone 1 S 8 S?
Ciprofloxacin 0.25 S 4 R
Ertapenem 0.5 S 0.5 S
Gentamicin 1 S 1 S
Imipenem 1 S 1 S
Levofloxacin 0.12 S 8 R
Nitrofurantoin 16 S 16 S
Piperacillin/ 4 S 128 R
tazobactam
Tobramycin 1 S 16 R
Trimethoprim/ 20 S 320 R
sulfamethoxazole
Tille: Bailey & Scott’s Diagnostic Microbiology, 14th Edition
Chapter 19: Enterobacteriaceae

Answers to Case Studies and Review Questions

Case Studies

Case Study 19-1

1. Yersinia organisms are characterized by having negative reactions at 35°C that are
positive at 25°C. Both motility and the VP reaction are positive for this organism at 25°C
but negative at 35°C. However, many strains are not positive for either reaction and
require serologic tests to identify.
2. Yersinia species are very slow-growing organisms, and the additional incubation
allowed expression of the enzymes that changed the reactions. In addition, Yersinia
organisms prefer to grow at low temperatures and produce more enzymes at 25°C.
3. The plague bacillus is not motile and is urease-negative. Yersinia pseudotuberculosis is
motile at 25°C and urease-positive; however, not all strains produce positive reactions.
4. Immediately notifying the physician, the local epidemiologist, and the local health
department is imperative. Even if the gram-negative rod is not ultimately diagnosed as
the plague bacillus, the life-threatening nature of this infection prompts the institution of
appropriate antimicrobial therapy and infection control action to prevent human-to-
human transmission.

Case Study 19-2

1. Ingestion of contaminated meat or handling of the livestock.


2. Enterotoxigenic Escherichia coli most likely contracted from ingesting raw hamburger.
3. The patient experienced dehydration and diarrhea, but suffered only a mild hemolysis.
The patient was extremely fortunate; the patient’s condition did not progress to acute
renal failure consistent with hemolytic uremic syndrome (HUS).

Review Questions

Multiple Choice

1. b; 2. b; 3. a; 4. a; 5. d; 6. b; 7. c; 8. d; 9. b

Matching

e, h, i, b, o, f, j, a, l, d, k, m, c, g, n

Short Answer

Isolate #1 wild strain of E. coli, negative extended spectrum β-lactamase (ESBL) test, no

Copyright © 2017, Elsevier Inc. All Rights Reserved.


Answers to Case Studies and Review Questions 19-2

corrections required. Isolate #2, positive ESBL test, corrections required to ceftazidime,
ceftriazone, and cefepime; sensitive corrected to resistant; cefazolin already resistant,
based on data provided.

Copyright © 2017, Elsevier Inc. All Rights Reserved.


CHAPTER 20 Acinetobacter, Stenotrophomonas, and Other Organisms 363.e1

Chapter Review
1. Which of the following are the third and fourth most com- 6. Which organisms are plump coccobacilli that may resist
mon gram-negative bacilli found in clinical specimens? decolorization and may be mistaken for Neisseria spp.?
a. Pseudomonas spp. and Acinetobacter spp. a. Acinetobacter spp.
b. Acinetobacter spp. and S. maltophilia b. Bordetella spp.
c. B. parapertussis and S. maltophilia c. Stenotrophomonas sp.
d. B. parapertussis and Pseudomonas spp. d. Burkholderia sp.
2. Which of the following patients are at higher risk of 7. Which test is used to differentiate asaccharolytic, nitrate-
Acinetobacter infection? reducing strains of Acinetobacter?
a. Patients in burn care units a. Oxidase
b. Patients in intensive care units b. Catalase
c. Patients who have received multiple antibiotics c. Motility
d. All of the above d. Acinetobacter transformation test
3. Which organism is occasionally found in the respiratory 8. True or False
tract of patients with cystic fibrosis? _____ The oxidase test can be used to differentiate
a. S. maltophilia between S. maltophilia and B. cepacia.
b. Bordetella holmesii _____ Acinetobacter species are divided into hemolytic
c. Bordetella parapertussis and nonhemolytic strains.
d. Burkholderia gladioli _____ Acinetobacter and S. maltophilia are susceptible to
4. The primary drug of choice for S. maltophilia is: a wide array of antimicrobials, making therapy
a. Trimethoprim-sulfamethoxazole quite easy.
b. Piperacillin 9. Matching: Match each term with the corresponding
c. Tobramycin description.
d. Levofloxacin _____ auxotrophic a. maltose loving
5. Which organism has large, smooth, glistening colonies _____ Pseudomonas b. unable to synthesize
with a lavender pigment and smells like ammonia? _____ maltophilia nutrients
a. Acinetobacter baumannii _____ Acinetobacter c. rarely found in
b. Pseudomonas luteola _____ Bordetella holmesii humans
c. Stenotrophomonas maltophilia d. produces yellow
d. None of the above colonies on
blood agar
e. nosocomial infection
genomospecies
Tille: Bailey & Scott’s Diagnostic Microbiology, 14th Edition
Chapter 20: Acinetobacter, Stenotrophomonas, and Other Organisms

Answers to Case Studies and Review Questions

Case Studies

Case Study 20-1

1. A positive catalase test and negative tests for oxidase and hanging drop motility will
separate this species from a number of other genera of asaccharolytic gram-negative rods.
If the isolate had been able to oxidize glucose, then these tests would be useful to
separate the organism from Pseudomonas luteola and Pseudomonas oryzihabitans,
which, unlike Acinetobacter, have a yellow pigment.
2. The nitrate test will separate CDC group NO-1 (Pseudomonas spp.), which is positive,
from asaccharolytic Acinetobacter, which do not convert nitrate to nitrite. However, in
this case, the test is not necessary, because CDC group NO-1 is often found in dogs and
cats or associated with ambulatory dialysis infections, and this patient has not left the
hospital. Some species of Bordetella (Chapter 24) are also asaccharolytic and oxidase-
and nitrate-negative, but they are either motile or urease-positive. The commercial system
would have detected the positive urease. Bordetella holmesii, previously referred to as
CDC group NO-2, should also be separated from asaccharolytic Acinetobacter. It grows
poorly on MacConkey agar, is often catalase-negative or weakly positive, and has a
brown soluble pigment on Mueller-Hinton agar.
3. Acinetobacter organisms are endogenous microbiota of the skin. This patient probably
acquired the organism from either his own skin or that of his caregivers. Acinetobacter
baumannii, a glucose-oxidizing member of this genus, is the third most common cause of
gram-negative infections in infants and has been responsible for serious nosocomial
spread of resistant strains in neonatal intensive care units. Its association with catheter
infections is rare.
4. An organism that can produce acid from glucose only in the presence of oxygen is a
glucose-oxidizing gram-negative rod. If it can produce acid from glucose without the
presence of oxygen, then it is a glucose-fermenting gram-negative rod. This test is
performed by inoculating two tubes of oxidation-fermentation medium, and overlaying
one with oil to prevent the absorption of oxygen in the medium (Figure 12-35).
5. The HACEK (AACEK) group (Chapter 30 for the list of bacteria) causes serious
infections. These bacteria are glucose-fermenters, do not grow in an oxidative-
fermentative (OF) medium, and are frequently misidentified by commercial identification
systems. They often need rabbit serum or other enrichment to grow in a tube medium,
giving rise to falsely negative nitrate and glucose fermentation results. One way to detect
glucose fermentation in these bacteria is to add a drop of rabbit serum to Andrade’s
glucose (Procedure 12-16). Another way is to use a rapid commercial system designed to
detect preformed enzymes of these fastidious microorganisms.

Review Questions

Copyright © 2017, Elsevier Inc. All Rights Reserved.


Answers to Case Studies and Review Questions 20-2

Multiple Choice

1. b; 2. d; 3. d; 4. a; 5. c; 6. a; 7. d

True/False

T, F, F

Matching

b, d, a, e, c

Copyright © 2017, Elsevier Inc. All Rights Reserved.


CHAPTER 21 Pseudomonas, Burkholderia, and Similar Organisms 375.e1

Chapter Review
1. Which of the following has a Gram stain morphology 9. PC agar contains crystal violet, bile salts, and polymyx-
that resembles safety pins? in B to:
a. B. diminuta a. Inhibit gram-positive organisms
b. B. mallei b. Inhibit slow-growing gram-negative organisms
c. P. pseudomallei c. Create a color change from red to yellow
d. Ralstonia d. Create a color change from blue to yellow
2. Which of the following may not grow on MacConkey 10. Which of the following is a valid testing method for
agar? P. aeruginosa?
a. P. aeruginosa a. Disk diffusion
b. B. vesicularis b. Broth dilution
c. B. cepacia c. Agar dilution
d. R. pickettii d. All of the above
3. Which of the following is the key pathogen that infects 11. True or False
the lungs of patients with cystic fibrosis? _____ BCSA, PC, or OFPBL should be used for
a. B. cepacia patients with cystic fibrosis.
b. B. pseudomallei _____ P. aeruginosa can be distinguished from other
c. P. fluorescens pseudomonads by growing at 25°C.
d. P. aeruginosa _____ B. cepacia should be suspected whenever a
4. Which organism should not be tested for antimicrobial nonfermentative organism that decarboxylates
susceptibility using the disk diffusion testing method? lysine is encountered.
a. P aeruginosa _____ Isolates of B. cepacia from cystic fibrosis patients
b. B. cepacia complex can be accurately identified by most commercial
c. B. mallei systems based on phenotypic characteristics and
d. S. maltophilia require no additional testing for confirmation.
5. All of the following are true of most Pseudomonas and _____ Isolates of Brevundimonas, Burkholderia, and
Burkholderia spp. except: Ralstonia spp. from clinical specimens are likely
a. Nitrate negative to be involved in acute infections of hospitalized
b. Oxidase positive patients.
c. Grows on MacConkey agar 12. Matching: Match each term with the corresponding
d. Oxidizes glucose description.
6. Which factor contributes to the pathogenicity of P. aeru- _____ Ashdown a. black, necrotic cutaneous
ginosa by inhibiting protein synthesis? _____ Ecthyma papules
a. Exotoxin A gangrenosum b. 30°C to 37°C optimal
b. Pyoverdin _____ glanders temperature for growth
c. Alginate _____ MALDI-TOF c. media used to isolate
d. Exoenzyme S MS B. cepacia complex
7. Which factor contributes to the formation of biofilms by _____ mesophilic d. media used to isolate
P. aeruginosa? _____ melioidosis B. pseudomallei
a. Exotoxin A _____ OFPBL e. B. mallei infection
b. Pyoverdin _____ ribotyping f. B. pseudomallei
c. Alginate infection
d. Exoenzyme S g. rRNA gene fingerprint
8. A reliable identification scheme for P. aeruginosa shows for species identification
all of the following except: h. peptide fingerprint for
a. Oxidase positive species identification
b. No growth at 37°C
c. Production of bluish-green pigment on Mueller-
Hinton agar
d. Triple sugar iron slant with an alkaline/no change
reaction
Tille: Bailey & Scott’s Diagnostic Microbiology, 14th Edition
Chapter 21: Pseudomonas, Burkholderia, and Similar Organisms

Answers to Case Studies and Review Questions

Case Studies

Case Study 21-1

1. Pseudomonas aeruginosa–mucoid strains, Alcaligenes xylosoxidans, Burkholderia


cepacia, and Stenotrophomonas maltophilia are frequently found in respiratory
specimens from patients with cystic fibrosis (CF).
2. P. aeruginosa with mucoid morphologic characteristics is typical of patients with CF.
It is not clear which lung factors in this patient population promote the expression of the
mucoid variant, but it is a hallmark of the disease. Often these strains are unable to
produce the characteristic pigment of P. aeruginosa.
3. The agents of infection are extremely slow growing. In a rapid test, their growth could
be inhibited, masking their true resistance.
4. The disk test can be incubated for 24 hours without loss of sensitivity. It allows testing
of drugs that may not be routinely tested in the laboratory but are needed to treat patients
who have acquired drug-resistant strains. The disk test can aid in the detection of mixed
cultures, which are often seen in this patient population.
5. B. cepacia is colistin-resistant, and S. maltophilia can also be resistant. They both are
lysine-positive and ferment maltose, but only B. cepacia ferments mannitol. S.
maltophilia hydrolyses DNA (see Figure 12-14 and is usually susceptible to
sulfamethoxazole-trimethoprim but is intrinsically resistant to imipenem. Ralstonia
pickettii and other Burkholderia organisms can be confused with B. cepacia. Because the
isolation of B. cepacia labels a patient with CF as contagious to other patients with CF, it
is important that the identification be confirmed.
6. Because patients with CF are uniformly infected with Pseudomonas, the smear will
show gram-negative rods. The important information for the physician caring for the
patient with CF is the susceptibility of the gram-negative isolates, not the rapid detection
of their presence in a smear.

Case Study 21-2

1. Yes, an elevated A1c level indicates uncontrollable diabetes along with the elevated
glucose. His additional chemistry results may indicate some additional systemic problem
associated with an elevated WBC count related to the lesion.
2. The Gram stain reveals a gram-negative rod with numerous WBCs indicating an
infection. The organism should be cultured. Serious wounds are often the result in
patients because of reduced circulation and may become systemic. The patient should
also be drawn for blood cultures to check for a systemic infection.
3. The patient is likely infected with a Pseudomonas spp., based on the bright green
purulent exudate and Gram-stain presentation. The wound should be drained and cleaned.

Copyright © 2017, Elsevier Inc. All Rights Reserved.


Answers to Case Studies and Review Questions 21-2

The resulting culture should be examined for susceptibility to begin appropriate antibiotic
therapy.

Review Questions

Multiple Choice

1. c; 2. b; 3. d; 4. c; 5. d; 6. a; 7. c; 8. b; 9. a

True/False

T, F, T, F, F

Matching

d, a, e, h, b, f, c, g

Copyright © 2017, Elsevier Inc. All Rights Reserved.


CHAPTER 22 Achromobacter, Rhizobium, Ochrobactrum, and Similar Organisms 383.e1

Chapter Review
1. Which of the following nonfermentative gram-negative 4. Matching: Match each term with the corresponding
bacilli are most commonly associated with clinical infections? description.
a. Ochrobactrum spp., Shewanella algae _____ R. radiobacter a. 20°C-25°C
b. Paracoccus yeei, Shewanella putrefaciens _____ S. algae b. H2S production
c. Ochrobactrum spp., Rhizobium radiobacter _____ P. yeei c. donut
d. Rhizobium radiobacter, Paracoccus yeei, Shewanella _____ S. putrefaciens d. halophilic
algae _____ Psychrobacter spp. e. curved rods
2. This nonfermentative gram negative rod is capable of _____ OFBA-1 f. acid reaction in OF tube
H2S production: _____ CDC group O-3 g. capsule production
a. Alcaligenes xylosoxidans
b. Shewanella algae
c. Ochrobactrum anthropi
d. Paracoccus yeei
3. True or False
____ These organisms are typically associated with high
infection rates in hospital patients.
____ All organisms in this group are associated with
environmental contamination and infections in
immunocompromised patients.
____ Antibiotic susceptibility patterns are easily com-
pleted for any organism that is cultivatable on stan-
dard laboratory media.
Tille: Bailey & Scott’s Diagnostic Microbiology, 14th Edition
Chapter 22: Achromobacter, Rhizobium, Ochrobactrum, and Similar Organisms

Answers to Case Studies and Review Questions

Review Questions

Multiple Choice

1. c; 2. b

True/False

F, T, F

Matching

g, c, b, a, f, e, d

Copyright © 2017, Elsevier Inc. All Rights Reserved.


CHAPTER 23 Chryseobacterium, Sphingobacterium, and Similar Organisms 389.e1

Chapter Review
1. Infection with which organism is associated with a high 4. True or False
mortality rate in neonates? ____ The organisms discussed in this chapter are easily
a. S. multivorum identifiable by their yellow pigmentations.
b. E. meningoseptica ____ M. odoratus is capable of nitrate reduction.
c. M. odoratus ____ Isolation of Weeksella virosa from a female's urinary
d. Chryseobacterium spp. specimen always requires antibiotic treatment, as
2. A female presents to the emergency department com- with any other prominent gram-negative rod.
plaining of frequency and burning on urination. A urine 5. Matching: Match the following terms to the corresponding
sample reveals a fruity smelling, gram-negative rod that is description or term.
oxidase, catalase, and urease positive and indole negative. _____ Myroides spp. a. animal bites
The organism is most likely: _____ Elizabethkingia sp. b. flexirubin
a. S. spiritivorum _____ Chryseobacterium c. fruity odor
b. E. coli indologenes d. beta-lactamase
c. P. aeruginosa _____ Weeksella virosa e. genitourinary
d. M. odoratus _____ Bergeyella f. sphingophospholipids
3. A gram-negative, slightly curved, long bacilli is isolated zoohelicum
from a female with respiratory symptoms that include _____ Sphingobacterium
dyspnea and cough. She recently was diagnosed with multivorum
asthma and had been prescribed breathing treatments,
which she administered at home using a humidified in-
haler. What type of specimen should be collected and
what media should be used to isolate the organism?
a. Blood and sputum; MacConkey and blood agar
b. Sputum; blood and chocolate agar
c. Blood and sputum; blood culture broth, chocolate
and blood agar
d. Blood; blood culture broth
Tille: Bailey & Scott’s Diagnostic Microbiology, 14th Edition
Chapter 23: Chryseobacterium, Sphingobacterium, and Similar Organisms

Answers to Case Studies and Review Questions

Case Studies

Case Study 23-1

1. In the differential are Weeksella virosa (a normal inhabitant of the genitourinary tract
primarily in primarily females), Bergeyella zoohelcum, a number of CDC group II
species, Chryseobacterium spp., Elizabethkingia meningoseptica, and Empedobacter
brevis. All of these genera are oxidase- and catalase-positive. Some have yellow
insoluble pigments.
2. Oxidation of mannitol in OF medium will definitively separate E. meningoseptica from
the Chryseobacterium spp. and the related species in the differential. Colonies frequently
lack the distinct yellow pigment, or, if present, it can be pale yellow. W. virosa and B.
zoohelcum are asaccharolytic and thus unable to produce acid even from glucose. The
patient’s isolate was identified as E. meningoseptica.
3. The failure of antimicrobial agents to eradicate the bacteria is believed to be the
bacteria’s resistance to usual agents. Conflicting results are found in the literature,
especially when disk methods are used. Only qualified persons in a reference laboratory
that use a CLSI minimum inhibitory concentration (MIC) method should perform the
testing. Drugs tested should include trimethoprim-sulfamethoxazole, rifampin, quinolone,
vancomycin, erythromycin, and minocycline. Many of these agents are not appropriate
for neonates, making treatment difficult.

Review Questions

Multiple Choice

1. b; 2. d; 3. c

True/False

F, T, F

Matching

c, d, b, e, a, f

Copyright © 2017, Elsevier Inc. All Rights Reserved.


CHAPTER 24 Alcaligenes, Bordetella (Nonpertussis), Comamonas, and Similar Organisms 398.e1

Chapter Review
1. Which organism grows better under cooler temperatures 6. The organisms presented in this chapter are grouped
(25°C) than in ambient air? together because they are all predominantly:
a. Pseudomonas alcaligenes a. Capable of growth on MacConkey agar, oxidase
b. Psychrobacter spp. positive, non–glucose utilizers
c. Comamonas spp. b. Capable of growth on MacConkey agar, oxidase
d. Cupriavidus pauculus negative, non–glucose utilizers
2. No mode of transmission has been identified for which c. Capable of growth on MacConkey agar, oxidase posi-
organism? tive, non–lactose fermenters
a. A. denitrificans d. Capable of growth on MacConkey agar, oxidase
b. B. pertussis negative, non–lactose fermenters
c. B. holmesii 7. Matching: Match each term with the corresponding term
d. B. bronchiseptica or phrase.
3. The organisms in this chapter are difficult to identify. _____ Bordetella a. mucoid
Strategies for identification should include: bronchiseptica b. polar flagella
a. Biochemical tests _____ Oligella spp. c. close contact with
b. Growth on MacConkey agar _____ Alcaligenes animals
c. Flagella staining faecalis d. urinary tract infection
d. A and C _____ Myroides spp. e. coccid, rods or filaments
e. All of the above _____ Roseomonas spp. f. feather-edged colonies
4. Which group of organisms is the only indole-positive, _____ Achromobacter g. spreading colonies
nonmotile species included in this chapter? spp. h. peritrichous flagella
a. Cupriavidus pauculus _____ Comamonas spp. i. broad, gram-negative
b. Oligella ureolytica _____ P. phenylpyruvicus rods
c. CDC group IIg _____ CDC group IIg
d. Psychrobacter phenylpyruvicus
5. Indole production is typically characterized by the for-
mation of a pink color on addition of Kovac’s reagent;
which of the following organisms is unique in its re-
sponse to Kovac’s reagent?
a. Achromobacter xylosoxidans produces no color.
b. Rhodococcus spp. produce a red color.
c. Roseomonas spp. produce a bright magenta color.
d. Delftia acidovorans produces an orange color.
Tille: Bailey & Scott’s Diagnostic Microbiology, 14th Edition
Chapter 24: Alcaligenes, Bordetella (Nonpertussis), Comamonas, and Similar
Organisms

Answers to Case Studies and Review Questions

Case Studies

Case Study 24-1

1. Spot indole, hanging drop motility, and 2-hour urease–phenylalanine deaminase (PDA)
tests are helpful. The urease-PDA test is purchased as a disk. Heavy growth of the
organism is emulsified in 1 mL of saline, and the disk is added. After 2 hours of
incubation, the tube is observed for the pink color of urea hydrolysis. If it is positive, then
a drop of 10% ferric chloride is added to detect the dark green color of a positive PDA
test.
2. Myroides and Psychrobacter immobilis are nonmotile. Oligella ureolytica and
Psychrobacter phenylpyruvicus are PDA-positive. Bordetella bronchiseptica and
Cupriavidus pauculus (formerly CDC IVc-2 and includes Ralstonia spp.; see Chapter 21)
are motile and urease-positive but PDA-negative. C. pauculus is PYR-positive. This
isolate was negative for PYR and was identified as B. bronchiseptica, a pathogen that
causes “kennel cough” in dogs. As further confirmation of the rapid identification, tube
biochemical tests were inoculated. B. bronchiseptica is asaccharolytic, does not produce
H2S, and is nitrate-positive with no gas.
3. Brucella spp. Brucella are oxidase-, catalase-, and urease-positive. The genus does not
grow well on MacConkey agar. The importance of performing rapid tests to screen for
this agent cannot be overemphasized. Whenever all three of these tests are positive and
the isolate is not growing on MacConkey agar, the microbiologist should begin to
perform all testing in a biological safety cabinet. Laboratory-acquired infections with this
pathogen have been documented both from the respiratory route and from direct skin
contact. This pathogen is also on the list of potential bioterrorist agents.
4. Because the isolate took 48 hours to form colonies, the laboratory could have missed
the identification, either because the microbiologists did not incubate the plates for 48
hours or because they did not recognize this tiny colony as different from the normal oral
microbiota that was on the plate.

Review Questions

Multiple Choice

1. b; 2. c; 3. e; 4. c; 5. d; 6. a

Matching

c, d, f, g, a, h, b, i, e

Copyright © 2017, Elsevier Inc. All Rights Reserved.


CHAPTER 25 Vibrio, Aeromonas, and Similar Organisms 406.e1

Chapter Review
1. A patient presents with diarrhea after spending 2 weeks 4. Matching: Match each term with the corresponding term
in Haiti after the country’s devastation by an earthquake. or description.
A stool specimen is collected and inoculated to enrich- _____ halophilic a. ear infections
ment broth before subculturing to TCBS. After 48 hours _____ Zot b. cellulitis and abscess
of incubation on TCBS, no growth is identified on the _____ C. violaceum formation
media. What should the laboratory scientist do next? _____ Aeromonas spp. c. salt loving
a. Request a new specimen. _____ V. alginolyticus d. enterotoxin
b. Run quality control organisms to check the integrity _____ V. cholerae e. gastroenteritis and
of the TCBS media. endocarditis
c. Report the culture as no growth with a comment that f. profuse watery diarrhea
indicates the organism may be viable but noncultur- 5. Short Answer
able, and the result does not rule out the presence of (1) Describe the string test and how it is used to differ-
an infection. entiate Vibrio spp. from Aeromonas spp.
d. Report all cultures as no growth. (2) What simple biochemical test can be used to differ-
2. A stool specimen is submitted for culture. The results are: entiate V. cholerae, V. mimicus, and Aeromonas spp.
beta-hemolytic on blood agar, NLF on MacConkey, oxi- from the other organisms discussed in this chapter?
dase positive, bull’s-eye appearance on CIN agar. This (3) Explain the chemical principle for the selective and
organism is most likely: differential properties of TCBS.
a. A. hydrophilia
b. Y. enterocolitica
c. C. violaceum
d. G. hollisae
3. A suspected isolate of Vibrio spp. is isolated from a young
child with diarrhea. The organism is identified as a
curved, gram-negative rod, oxidase and lactose positive,
sucrose negative, that produces yellow colonies on TCBS
and is NaCl tolerant. This organism is most likely:
a. V. mimicus
b. V. furnissii
c. V. cholerae
d. V. fluvialis
Tille: Bailey & Scott’s Diagnostic Microbiology, 14th Edition
Chapter 25: Vibrio, Aeromonas, Plesiomonas shigelloides, and Chromobacterium
violaceum

Answers to Case Studies and Review Questions

Case Studies

Case Study 25-1

1. This isolate could be either Aeromonas or Vibrio. The string test (Figure 25-4), growth
on thiosulfate citrate bile salts sucrose (TCBS) agar, O129 disk susceptibility, or salt
tolerance can be used to separate these genera. The organism produced yellow colonies
on TCBS agar because it was positive for sucrose. A commercial system accurately
identified the bacterium as Vibrio alginolyticus.
2. The most common Vibrio found in the ocean is V. alginolyticus. Pseudomonas
aeruginosa is most often responsible for external ear infections from freshwater (called
“swimmer’s ear”), and V. alginolyticus is the common pathogen responsible for ear
infections from swimming in saltwater (Chapter 71 Case Study).
3. Both genera share a number of biochemical reactions. Vibrio fluvialis is a pathogen
with serious consequences but shares a biochemical profile similar to Aeromonas caviae,
a common species found in stool cultures. Without performing the tests that separate
these genera, which are not found in commercial systems, misidentifications can easily be
made.
4. To perform the disk test successfully, the initial inoculum equivalent to the 0.5
McFarland turbidity standard should be made in saline, not water. This step will ensure
that the organism will grow well and demonstrate the correct resistance pattern. V.
alginolyticus is resistant to Polymyxin B, which is commonly found in eardrops used to
treat “swimmer’s ear.” Identification of the pathogen with susceptibility testing is needed
for proper treatment.

Review Questions

Multiple Choice

1. c; 2. b; 3. a

Matching

c, d, b, e, a, f

Short Answer

1. See Figure 25-4. Organisms are emulsified in 0.5% sodium deoxycholate, which lyses
Vibrio cells, but not Aeromonas spp. cells. Cell lysis releases DNA resulting in “string”

Copyright © 2017, Elsevier Inc. All Rights Reserved.


Answers to Case Studies and Review Questions 25-2

formation when the mixture is pulled up with an inoculating loop.


2. Salt tolerance.
3. TCBS contains 1% NaCl, and bile salts that inhibit (select against) gram-positive
organisms and sucrose for differentiation based on carbohydrate metabolism. Colonies
capable of fermenting sucrose appear yellow (acid production, altering pH, color
indicator bromothymol or thymol blue).

Copyright © 2017, Elsevier Inc. All Rights Reserved.


CHAPTER 26 Sphingomonas paucimobilis and Similar Organisms 413.e1

Chapter Review
1. Of all the bacteria discussed in this chapter, which one 4. True or False
has been most highly implicated in disease? _____ Growth on 5% sheep blood, chocolate, and
a. A. facilis MacConkey agar plates is a common trait of
b. S. mizutaii Sphingomonas spp.
c. S. paucimobilis _____ Most bacteria discussed in this chapter are com-
d. CDC group IIc mon causes of pathogenicity.
2. Initial clues of the presence of this group of bacteria for _____ Although antimicrobial susceptibility results of
clinical laboratorians are: bacteria discussed in this chapter have been re-
a. They produce yellow pigment, do not grow on ported in the literature, there is currently no vali-
MacConkey agar, and oxidize glucose. dated method available.
b. They produce no pigment, do not grow in thioglycollate 5. Matching: Match the bacterium with the appropriate test
broth, and oxidize glucose. result. (Results can be used more than once.)
c. They produce tan/buff pigment, do not grow on
_____ S. paucimobilis a. yellow pigment
blood agar, and oxidize xylose.
d. They produce yellow pigment, do not grow on blood _____ S. multivorum b. oxidize glucose
agar, and ferment glucose. _____ CDC group IIc c. hydrolyze esculin
3. Identification approaches useful for speciation of Sphin- d. produce urease
gomonas are: e. produce DNase
a. H2S production
b. Citrate utilization
c. DNase production
d. a and c
e. a, b, and c
Answers to Case Studies and Review Questions 26-2

1. Because the organism is nonmotile, CDC groups IIc, IIe, IIh, IIi, O-2, and
Sphingobacterium are suspect.
2. The following table displays test results of importance in differential bacterial
identification.

Pigmen Sucrose Esculin Urease Cell Xylose


t Oxidatio Hydrolys Morpholog Oxidatio
n is y n
CDC group IIc Tan or Positive Positive Negativ Short, Negative
buff; e straight
colonies rods with
are thickened
sticky ends and
thin centers
CDC group IIe No Negative Negative Negativ Short, Negative
e straight
rods with
thickened
ends and
thin centers
CDC group IIh No Negative Positive Negativ Short, Negative
e straight
rods with
thickened
ends and
thin centers
CDC group IIi Yellow Positive Positive – Short, Positive
straight
rods with
thickened
ends and
thin centers
CDC group O-2 Yellow Positive Variable – Short, Negative
to straight
orange rods with
thickened
ends and
thin centers
Sphingobacteriu Yellow – Positive Positive Rods –
m multivorum
Sphingobacteriu Yellow – Positive Positive Rods –
m spiritivorum
S. mizutaii Yellow Positive Positive Negativ Rods with –
e thickened
ends and
thin centers

Copyright © 2017, Elsevier Inc. All Rights Reserved.


Answers to Case Studies and Review Questions 26-3

3. Validated susceptibility testing does not exist. Many literature reports identify
susceptibility results using E-test performed on blood agar plates.

Review Questions

Multiple Choice

1. c; 2. a; 3. e

True/False

F, F, T

Matching

1. a, b, c, e
2. a, b, c, d
3. b, c

Copyright © 2017, Elsevier Inc. All Rights Reserved.


Tille: Bailey & Scott’s Diagnostic Microbiology, 14th Edition
Chapter 26: Sphingomonas paucimobilis and Similar Organisms

Answers to Case Studies and Review Questions

Case Studies

Case Study 26-1

1. CDC group O-2, Sphingobacterium mizutaii, Sphingomonas paucimobilis, and


Sphingomonas parapaucimobilis
2. The following table displays test results of importance in differential bacterial
identification.

Glucose Esculin Cell Xylose


Motility Oxidation Hydrolysis Citrate Morphology Oxidation
CDC group O-2 Usually Variable Usually – Short, Negative
motile positive straight rods
with
thickened
ends and
thin centers
S. mizutaii Nonmotile Positive Positive – Rods with Delayed
thickened positive
ends and
thin centers
S. paucimobilis Nonmotile Positive Positive Negative Medium Positive
in motility size, straight
medium; rods with
incubated at single polar
37C but flagellum
motile at
18C to
22C;
motile on
wet mount
S. Positive Positive Positive Positive Medium Positive
parapaucimobilis size, straight
rods

3. Validated susceptibility testing does not exist. Many literature reports identify
susceptibility results using E-test performed on blood agar plates.

Case Study 26-2

Copyright © 2017, Elsevier Inc. All Rights Reserved.


CHAPTER 27 Moraxella 418.e1

Chapter Review
1. Which of the following species of the gram-negative, 7. Matching
nonfermenting group of bacteria is considered normal _____ Moraxella a. Gram-variable cocci in pairs
oropharyngeal flora in cats and dogs and is typically iso- osloensis or short chains
lated from the bite wounds of these animals? _____ Moraxella canis b. Gram-negative coccobacilli
a. N. weaverii _____ Moraxella that may appear in chains
b. M. canis lacunata c. Gram-variable coccobacilli or
c. M. catarrhalis _____ Moraxella short, broad rod
d. both a and b lincolnii d. Gram-negative, medium-
e. both b and c _____ Neisseria length straight bacillus
2. Which of the following species of Moraxella is normal elongata e. Gram-negative coccobacilli
human microbiota inhabiting the urogenital tract? _____ Neisseria or short, straight rods
a. M. lacunata weaverii f. Gram-negative coccobacilli
b. M. lincolnii or medium-size rod
c. M. osloensis 8. Short Answer
d. M. atlantae (1) Explain the procedure used to differentiate true dip-
3. Which of the following species of the Neisseria group of lococcic from bacillary forms within the genera of
bacteria is able to utilize glucose? bacteria Moraxella and Neisseria.
a. N. elongata subsp. glycolytica (2) Explain the significance of the isolation of a species
b. N. elongata subsp. nitroreducens of this group of bacteria, Moraxella and Neisseria, in
c. N. weaverii culture.
d. N. elongata subsp. elongata
4. Which of the species in the Moraxella group of bacteria
does not pit BA in culture?
a. M. nonliquefaciens
b. M. lincolnii
c. M. lacunata
d. M. atlantae
5. Which of the following species of Moraxella is able to
liquefy serum, causing depressions in the surface of
Loeffler’s serum agar slants?
a. M. nonliquefaciens
b. M. osloensis
c. M. lacunata
d. M. canis
6. True or False
_____ The colonial appearance of the bacteria N. elongata
(all subspecies) on BA is gray, translucent, smooth,
glistening colonies, which may also have a dry,
claylike consistency.
_____ Susceptibility testing is normally performed on
isolates of Moraxella and Neisseria spp.
_____ In the family of bacteria Moraxella, the biochemi-
cal characteristics are oxidase positive and catalase
negative.
_____ The test used to differentiate M. nonliquefaciens and
M. osloensis is acetate, because M. osloensis utilizes
acetate, whereas M. nonliquefaciens does not.
_____ Moraxella spp. and the elongated Neisseria spp.
grow well on 5% sheep blood, chocolate, and
MacConkey agar.
_____ The species of Moraxella most commonly associ-
ated with human infection, mostly respiratory, is
M. catarrhalis.
_____ The species of Moraxella known to cause conjunc-
tivitis is M. lacunata.

Tille 978-0-323-35482-0/00036
Tille: Bailey & Scott’s Diagnostic Microbiology, 14th Edition
Chapter 27: Moraxella

Answers to Case Studies and Review Questions

Case Studies

Case Study 27-1

1. From the description, this bacterium could be one of the microorganisms in the
HACEK (AACEK) group (Chapter 30). However, most bacteria of the HACEK
(AACEK) group are glucose-fermenters. Eikenella corrodens is the exception. The use of
Andrade’s glucose medium, supplemented with rabbit serum or plasma, has already been
discussed to grow these bacteria and to demonstrate their fermentation properties
(Chapter 20 Case Study). When this test is set up, a nitrate reduction test is also important
to separate E. corrodens from Neisseria elongata.
2. The most likely identification is that of N. elongata subspecies elongata. Because the
catalase test was negative, the isolate is not Neisseria weaverii. Because the nitrate was
negative, it is not E. corrodens (Chapter 28). However, this identification is based on
completely negative results, except for the positive oxidase reaction, colony morphology,
and Gram stain. Confirmation with at least one positive test should be performed. The
test used to confirm the identification is a positive nitrite reduction test.
3. To perform the test, nitrite must be the substrate. When the nitrate A and B reagents
are added to the uninoculated nitrite tube, a red color is produced from the reaction of the
reagents with nitrite. If the organism is inoculated to a tube of nitrite and the nitrite is
reduced, then no red color will be seen when the reagents are added. Incubation may have
to be extended; the red color will continue to appear until the organism reduces all the
nitrite in the tube. Most laboratories do not have this medium readily available, in which
case, confirmation should be performed at a reference laboratory.

Review Questions

Multiple Choice

1. d; 2. c; 3. a; 4. b; 5. c

True/False

T, F, F, T, F, T, T

Matching

c, a, f, b, e, d

Short Answer

Copyright © 2017, Elsevier Inc. All Rights Reserved.


Answers to Case Studies and Review Questions 27-2

1. Exposure to penicillin by streaking a blood agar plate and adding a 10-unit penicillin
disk to the first quadrant before overnight incubation at 35C. The presence of the
penicillin disk causes the coccoid forms of these bacteria to elongate to a bacillus
morphologic appearance, whereas those that are true coccoid forms will not elongate.
After overnight incubation, a Gram stain of the growth taken from around the edge of the
zone of inhibition demonstrates whether the isolate is a true coccus or has elongated.
Moraxella catarrhalis and most Neisseria spp. are true cocci and will not elongate in the
presence of penicillin.
2. Most exist as low-virulence, nonpathogenic forms. The three species that are
frequently isolated pathogens are M. catarrhalis, Neisseria gonorrhoeae, and Neisseria
meningitidis. Other than these three, the rest of the species rarely cause infection and
should be considered potential contaminants.

Copyright © 2017, Elsevier Inc. All Rights Reserved.


CHAPTER 28 Eikenella corrodens and Similar Organisms 423.e1

Chapter Review
1. Which of the following culture characteristics alert the 5. True or False
microbiologist to a possible infection resulting from ____ Methylobacterium is a bacterium normally found as
E. corrodens? human skin microbiota.
a. Rapid growth on 5% sheep blood, chocolate, and ____ Bergeyella zoohelcum can cause an opportunistic
MacConkey agar infection stemming from the wound of a dog or
b. Pitting of the agar and a characteristic chlorine bleach cat bite.
smell ____ Eikenella corrodens is a vacuolated, pale-staining,
c. Growth with alpha-hemolysis on 5% sheep blood short to medium-length bacillus, which resists
agar and no growth on MacConkey agar decolorization.
d. Limited growth on 5% sheep blood, chocolate, and ____ Eikenella corrodens readily utilizes glucose and other
MacConkey agar, but improved growth using BYCE carbohydrates and is catalase negative.
media ____ Methylobacterium spp. grow readily at an incuba-
2. From which of the following infections is E. corrodens tion temperature of 42°C.
most often isolated? ____ No special considerations are needed for the speci-
a. Urinary tract infections men collection and transportation for Eikenella
b. Sexually transmitted infections corrodens and similar organisms.
c. Middle ear infections ____ Eikenella corrodens will show limited growth in cul-
d. Human bite wound infections ture when using 5% sheep blood and chocolate
3. Which one of the following organisms grows best at agar but will grow readily on MacConkey agar.
an incubation temperature of 25°C, instead of 35°C to 6. Short Answer
37°C? (1) What organisms are in the HACEK (AACEK) group
a. Bergeyella and what medical conditions are associated with
b. Weeksella these bacteria?
c. E. corrodens (2) From what types of collection sites is the organism
d. Methylobacterium E. corrodens typically isolated?
4. The bacteria Weeksella and Bergeyella are biochemically (3) What factors prevent definitive treatment guidelines
similar. Which of the following test results is the correct for E. corrodens, Methylobacterium spp., Weeksella spp.,
result to differentiate the organisms? and Bergeyella spp.?
a. Weeksella is urease positive; Bergeyella is urease negative. (4) What test is used to distinguish between the organisms
b. Weeksella is urease negative; Bergeyella is urease positive. W. virosa and B. zoohelcum? What unusual characteris-
c. Weeksella is indole positive; Bergeyella is indole negative. tic do these organisms share that sets them apart from
d. Weeksella is indole negative; Bergeyella is indole positive. other nonfermentative bacteria?
Tille: Bailey & Scott’s Diagnostic Microbiology, 14th Edition
Chapter 28: Eikenella corrodens and Similar Organisms

Answers to Case Studies and Review Questions

Case Studies

Case Study 28-1

1. The organism in the culture was ornithine-positive. That, combined with its unique
odor, identifies it as Eikenella corrodens. For completeness, as with all isolates of gram-
negative rods, the spot indole was performed to confirm a negative reaction.
2. E. corrodens grows slowly on blood and chocolate agar. The organism will not grow
on MacConkey agar. The organism has a characteristic tendency to pit or corrode the
agar and exude a chlorine bleach odor. The organism is assachrolytic, oxidase positive,
catalase negative, reduces nitrate to nitrite, and hydrolyzes both ornithine and lysine. The
other organisms in this grouping are catalase positive.
3. The physicians were interested in finding a source of the infection. E. corrodens,
similar to other members of the HACEK (AACEK) group (Chapter 30), are found as
normal microbiota of the mouth and generally cause infections only when the mouth is in
contact with the vascular system. Most infections occur because of bleeding during dental
procedures.
4. This bacterium can cause infective endocarditis, as can the other members of the
HACEK (AACEK) group. These bacteria travel from the mouth through the bloodstream
and can lodge in the heart. Blood cultures are positive in most cases. Without treatment,
usually with a penicillin, the prognosis is poor. Because of the importance of detection,
the laboratory plays a critical role in the diagnosis of these infections.

Review Questions

Multiple Choice

1. b; 2. d; 3. d; 4. b.

True/False

F, T, F, F, F, T, F

Matching

e, d, a, c, b

Short Answer

1. Methylobacterium will use acetate, and Roseomonas will not; Roseomonas will grow at

Copyright © 2017, Elsevier Inc. All Rights Reserved.


Answers to Case Studies and Review Questions 28-2

42C, and Methylobacterium will not.


2. This organism is often found from infections resulting from a bite or lick wound and is
present as normal oral microbiota and can cause an opportunistic infection when wounds
or abscesses are exposed to the saliva from the mouth. It is also known to cause subacute
bacterial endocarditis.
3. The rarity with which these organisms are encountered and the lack of validated in
vitro susceptibility testing methods.
4. Urease is the test, Weeksella is urease-negative, and Bergeyella is urease-positive. Both
are indole-positive.

Copyright © 2017, Elsevier Inc. All Rights Reserved.


CHAPTER 29 Pasteurella and Similar Organisms 429.e1

Chapter Review
1. Into which genus have Haemophilus aphrophilus and 4. True or False
Haemophilus paraphrophilus been reclassified? _____ Pasteurella spp. infections in humans are most
a. Actinobacillus commonly of human origin.
b. Aggregatibacter _____ Commercial biochemical systems definitively iden-
c. Haemophilus tify unusual Pasteurella spp. with a high degree of
d. Pasteurella fidelity.
2. Which of the following characteristics is NOT seen with _____ Serodiagnosis is of little utility for the diagnosis
Pasteurella spp.? of infection caused by Pasteurella spp. and similar
a. Glucose fermentation organisms.
b. Positive oxidase reaction 5. Short Answer
c. Penicillin susceptibility Describe two testing methods that are useful for the
d. Luxurious growth on MacConkey agar identification of unusual Pasteurella spp. and related
3. Which of the following conditions is viewed as a risk factor organisms.
for systemic disease caused by Pasteurella multocida subsp.
multocida?
a. Liver cirrhosis
b. End-stage renal disease
c. Hyperlipidemia
d. Hereditary hemochromatosis
Tille: Bailey & Scott’s Diagnostic Microbiology, 14th Edition
Chapter 29: Pasteurella and Similar Organisms

Answers to Case Studies and Review Questions

Case Studies

Case Study 29-1

1. The organism is most likely Pasteurella multocida. The organism was incapable of
growth on selective enteric media (MacConkey Agar).
2. Most Pasteurella isolates are susceptible to oral antibiotics including penicillins,
fluroquinolones, and trimethoprim-sulfamethoxazole. However, in severe infections,
intravenous therapy is recommended in animal-bite injuries. Ampicillin-sulbactam is a
penicillin-derived antibiotic combined with a β-lactamase inhibitor (sulbactam).
Although Pasteurella is usually susceptible to penicillin-based antimicrobials and does
not produce β-lactamase, the empiric therapy with a β-lactamase inhibitor provides
coverage for a possible mixed infection prior to isolation of the organisms.
3. Cat Scratch Disease (CSD) also referred to as Cat Scratch Fever maybe associated with
bite wounds or scratches and even licks on open wounds. Although rare, CSD may begin
as a lesion that may lead to lymphadenopathy, headache, joint pain, and flu-like
symptoms. Severe cases may lead to sepsis or septic shock and renal failure if appropriate
therapy is not initiated. In this case, appropriate IV therapy was initiated resulting in an
improvement in the patient’s condition.

Review Questions

Multiple Choice

1. b; 2. d; 3. a.

True/False

F, F, T

Short Answer

1. Pasteurella species may be difficult to identify. Traditional biochemical tests include


oxidase, catalase, nitrate reduction urea, ornithine decarboxylase, mannitol, sucrose, and
maltose fermentation. However, a reference laboratory that can definitely identify the
species may be necessary. Reference laboratories may use gene sequencing or MALDI-
TOF MS. Large laboratories may also be able to speciate these isolates with successful
integration of MALDI-TOF MS technology.

Copyright © 2017, Elsevier Inc. All Rights Reserved.


CHAPTER 30 Actinobacillus, Kingella, Cardiobacterium, Capnocytophaga, and Similar Organisms 436.e1

Chapter Review
1. The organisms discussed in this chapter are transmitted 3. True or False
by all of the following except: _____ Actinobacillus spp. grow best in CO2 under dry
a. Droplet person to person conditions.
b. Animal bite _____ All the organisms discussed in this chapter grow
c. Sexual contact well in commercial blood culture systems.
d. Trauma _____ Specimens suspected of containing Capnocytoph-
2. A patient having a fever and chills came to the emergency aga spp. should be incubated for up to 7 days.
department. The patient reported having his wisdom 4. Matching
teeth removed 2 days prior. After 4 days of incubation, a _____ Actinobacillus spp. a. endocarditis
gram-negative bipolar staining rod was isolated from the _____ Kingella spp. b. septicemia
patient’s blood. The organism was catalase positive and _____ dysgonic c. hemolytic uremic
indole negative. The organism is most likely: _____ Capnocytophaga spp. syndrome
a. C. hominis _____ C. canimorsus d. osteoarthritis
b. A. actinomycetemcomitans _____ K. kingae e. slow growing
c. K. kingae f. meningitis
d. A. aphrophilus
Tille: Bailey & Scott’s Diagnostic Microbiology, 14th Edition
Chapter 30: Actinobacillus, Kingella, Cardiobacterium, Capnocytophaga, and Similar
Organisms

Answers to Case Studies and Review Questions

Case Studies

Case Study 30-1

1. The organism is a Capnocytophaga spp. that is part of the normal microbiota of the
mouth of humans. The patient is at risk for bacteremia because of her low WBC count
and her periodontal disease.
2. Few human pathogenic gram-negative rods are both catalase- and oxidase-negative and
grow in 5% carbon dioxide (CO2) under aerobic conditions. Among these, only
Capnocytophaga is a fusiform rod. Confirmation of the identification includes its
characteristics of being indole-negative, nonhemolytic, and often esculin-positive.
Identification to the species level is difficult and is not clinically important.
3. Capnocytophaga has historically been treated with β-lactam antibiotics, but currently
more than 75% of the organisms have been found to produce a β-lactamase. Testing for
the enzyme with the chromogenic cephalosporin test is sufficient to inform the physician
that treatment with a β-lactam drug will not be effective.

Review Questions

Multiple Choice

1. c; 2. b

True/False

F, F, T

Matching

f, a, e, b, c, d

Copyright © 2017, Elsevier Inc. All Rights Reserved.


CHAPTER 31 Haemophilus 445.e1

Chapter Review
1. All species of the genus Haemophilus require which of the 6. True or False
following for in vitro growth? _____ H. influenza can be found as normal microbiota
a. Nicotine adenine dinucleotide (NAD) of the upper respiratory tract of humans.
b. Cystine _____ H. influenza type a vaccine has been developed to
c. Hemin decrease infection in children.
d. A and C _____ Five percent sheep blood agar provides the factors
2. Which of the following Haemophilus spp. is an agent of a necessary for the growth of all Haemophilus spp.
sexually transmitted disease? _____ Haemophilus spp. are able to grow on MacConkey
a. H. parainfluenzae agar.
b. H. influenzae _____ Most strains of Haemophilus can grow anaerobi-
c. H. ducreyi cally and aerobically.
d. H. hemolyticus 7. Matching: Match the correct term with the appropriate
3. Which encapsulated type of H. influenzae is most description.
common? ____ hemin a. conjunctivitis
a. Type a ____ NAD b. V factor
b. Type b ____ H. influenza c. chancroid
c. Type c ____ H. aegypticus d. determines X factor
d. Type d ____ H. ducreyi requirement
4. All of the following organisms require X and V factors, ____ ALA-porphyrin test e. Staphylococcus streak
except: ____ satellite technique
a. H. influenzae phenomenon f. X factor
b. H. haemolyticus ____ acridine orange g. type b
c. H. influenzae biotype aegyptius h. detects smaller numbers
d. A. aphrophilus of organisms
5. Which of the following is an effective drug for treating
chancroid?
a. Ampicillin
b. Erythromycin
c. Amoxicillin
d. Tetracycline
Tille: Bailey & Scott’s Diagnostic Microbiology, 14th Edition
Chapter 31: Haemophilus

Answers to Case Studies and Review Questions

Case Studies

Case Study 31-1

1. The organism is in the genus Haemophilus, which is the most common cause of
epiglottitis. In 5% of the cases, endocarditis develops as a complication.
2. The organism is Aggregatibacter aphrophilus and is differentiated from Haemophilus
parainfluenzae by its ability to ferment lactose. For specimens from blood cultures,
fermentation testing is important to accurately identify Haemophilus and closely related
species such as A aphrophilus, to the species level.
3. Because the organism was not H. influenzae, which is another cause of epiglottitis,
family contacts did not have to receive prophylaxis with rifampin or other agents.
Isolation of A. aphrophilus, as well as H. influenzae, in blood cultures is generally
considered an indication of epiglottitis and warrants aggressive treatment. Isolation of H.
parainfluenzae and Haemophilus parahaemolyticus are less often associated with a
serious disease.

Review Questions

Multiple Choice

1. c; 2. c; 3. b; 4. d; 5. b

True/False

T, F, F, F, T

Matching

f, b, g, a, c, d, e, h

Copyright © 2017, Elsevier Inc. All Rights Reserved.


CHAPTER 32 Bartonella and Afipia 450.e1

Chapter Review
1. Humans acquire Bartonella infection in the following way: 5. Bartonella species can be detected by which of the
a. Arthropod-borne transmission following?
b. Rodents a. Warthin-Starry silver stain
c. Naturally b. PCR
d. A and C c. Immunofluorescence
2. Most Bartonella infections are thought to be: d. All of the above
a. Nosocomial infections 6. IFA testing for Bartonella has been known to cross-react
b. Zoonotic infections with:
c. Normal flora a. Chlamydia species
d. All of the above b. Coxiella burnetii
3. Bartonella is characterized by all of the following, except: c. H. influenzae
a. Gram negative d. A and B
b. Oxidase negative 7. Which of the following aid in Bartonella prevention for
c. MacConkey negative the immunocompromised patient?
d. Chocolate positive a. Vaccination
4. B. quintana has been known to cause: b. Avoiding contact with cats
a. Carrion disease c. Controlling flea infestation
b. Trench fever d. B and C
c. CSD
d. Lyme disease
Tille: Bailey & Scott’s Diagnostic Microbiology, 14th Edition
Chapter 32: Bartonella and Afipia

Answers to Case Studies and Review Questions

Case Studies

Case Study 32-1

1. Laboratories should have a policy that stipulates that if a bacterium is isolated from a
normally sterile site, even if the ordered test is for a specific agent, then the bacterium
should be identified. If the laboratory is unable to grow the organism, then the broth
bottle should be submitted to a reference laboratory.
2. The patient was homeless and slept on the ground where stray cats and dogs are
prevalent. He possibly was exposed to the bacterium via respiratory inhalation of
contaminated air and, with his poor health, was not able to clear it from his lungs.
3. Culture is rarely necessary because both diseases are self-limiting in healthy
individuals. For patients who are immunocompromised, especially those who are positive
for the human immunodeficiency virus (HIV), offering either culture or polymerase chain
reaction (PCR) or arranging to have specimens sent to a reference laboratory for testing is
probably advisable.

Review Questions

Multiple Choice

1. d; 2. b; 3. c; 4. b; 5. d; 6. d; 7. d

Copyright © 2017, Elsevier Inc. All Rights Reserved.


CHAPTER 33 Campylobacter, Arcobacter, and Helicobacter 460.e1

Chapter Review
1. Campylobacter are: 9. All of the following agars may be used to isolate Helico-
a. Small, curved, motile, gram-positive bacilli bacter except:
b. Small, curved, motile, gram-negative bacilli a. Campy-CVA
c. Small, curved, nonmotile, gram-negative bacilli b. Skirrow
d. Small, curved, nonmotile, gram-negative bacilli c. Thiosulfate-citrate-bile salts-sucrose (TCBS)
2. Campylobacter species produce the following syndromes d. Modified Thayer-Martin
in immunocompetent patients except: 10. H. pylori may be identified presumptively by which
a. Endocarditis positive tests?
b. Febrile systemic disease a. NO3, hippurate, catalase
c. Periodontal disease b. Oxidase, indoxyl acetate, NO3
d. Gastroenteritis c. Oxidase, catalase, rapid urea
3. C. jejuni and C. coli are usually transmitted by: d. NO3, catalase, growth at 25°C
a. Food 11. True or False
b. Milk _____ Hippurate hydrolysis can be used to differentiate
c. Water C. jejuni from all other Campylobacter species.
d. All of the above _____ Campylobacter causes febrile systemic disease, peri-
4. Which Campylobacter species has been recognized as odontal disease, and gastroenteritis in humans.
the most common causative agent of gastroenteritis in _____ Campylobacter is most commonly transmitted
the United States? via respiratory droplets.
a. C. lari _____ Stool specimens should be plated on MacConkey,
b. C. fetus Hektoen enteric (HE), and sorbitol-MacConkey
c. C. coli (SMAC) agar for optimal recovery of Campylo-
d. C. jejuni bacter species.
5. Which of the following has been recognized in postin- _____ Curved, microaerophilic, gram-negative rods
fection complications of a C. jejuni infection? showing strong urease activity are suggestive of
a. Guillain-Barré syndrome Campylobacter species.
b. Chronic pulmonary disease _____ H. pylori causes gastritis, peptic ulcer disease,
c. Encephalitis and gastric cancer.
d. Endocarditis 12. Matching: Match the correct term with the appropriate
6. Campylobacter species should be grown at what optimum description.
temperature? _____ microaerobic a. removal of the myelin
a. 25°C _____ Guillain-Barré sheath from a nerve
b. 37°C _____ Cary-Blair b. long, polar flagella
c. 42°C _____ seagull-winged c. requires less oxygen
d. None of the above _____ darting motility d. neurologic syndrome
7. A positive hippurate hydrolysis is a characteristic of: _____ H. pylori e. a selective agar
a. C. coli _____ Skirrow f. urea diagnostic test in
b. C. jejuni _____ demyelination broth
c. C. lari g. helix-shaped morphology
d. C. fetus h. transport medium
8. Campylobacter infection may be prevented by which of
the following?
a. Thoroughly cooking all foods
b. Pasteurized milk
c. Chlorinated water
d. All of the above
Tille: Bailey & Scott’s Diagnostic Microbiology, 14th Edition
Chapter 33: Campylobacter, Arcobacter, and Helicobacter

Answers to Case Studies and Review Questions

Case Studies

Case Study 33-1

1. Only Campylobacter jejuni is positive for hippurate among the strongly catalase-
positive Campylobacter spp.
2. Both disk testing and indoxyl acetate testing are needed to identify species that are
hippurate-negative. Campylobacter coli is positive and Campylobacter lari is negative
for indoxyl acetate. However, many other species are positive in this test. Only C. coli
and Campylobacter jejuni are positive and cephalothin resistant. Only C. lari is indoxyl
acetate–negative and cephalothin resistant. Thus the combination of both tests will
identify these three species, the most common stool pathogens.
3. The chicken was likely the source of the Campylobacter; cooking will, however, kill
the bacterium. If the cooked meat had been placed on the same surface that was used to
clean the uncooked meat, then it could have become contaminated after cooking.
Chickens harbor large numbers of Campylobacter organisms.
4. Previously, fluroquinolones (ciprofloxacin) were the antibiotics most frequently
prescribed for campylobacter infection. However, an increasing number of C. jejuni and
C. coli isolates in the United States are resistant to ciprofloxacin. When the organism was
initially discovered, part of the identification included susceptibility to nalidixic acid.
Because nalidixic acid is in the same family of drugs as ciprofloxacin, all ciprofloxacin-
resistant strains are resistant to nalidixic acid, making the test unusable in the
identification. Susceptibility tests for Campylobacter spp. are not standardized and not
routinely performed. C. jejuni and C. coli are susceptible to many antimicrobials
including macrolides, tetracyclines, aminoglycosides, and quinolones. Erythromycin is
the drug of choice for patients suffering with severe gastroenteritis.

Review Questions

Multiple Choice

1. b; 2. a; 3. d; 4. d; 5. a; 6. c; 7. b; 8. d; 9. c; 10. c

True/False

T, T, F, F, F, T

Matching

c, d, h, g, b, f, e, a

Copyright © 2017, Elsevier Inc. All Rights Reserved.


CHAPTER 34 Legionella 468.e1

Chapter Review
1. Legionella can be spread by all of the following except: 8. Which of the following is not a characteristic of
a. Cooling towers Legionella?
b. Person-to-person contact a. Faintly staining, thin, gram-negative bacilli
c. Lakes b. Requires iron and L-cysteine supplements
d. Humidifiers c. Growth enhanced by microaerophilic conditions
2. All of the following are primary manifestations of d. Medium buffered to pH 6.9 for optimal growth
legionnaires’ disease except: 9. All of the following are true of Legionella except:
a. Trench fever a. Thrives at warmer temperatures
b Pontiac fever b. Can survive up to 5 years in water
c. Endocarditis c. Has been isolated in rivers and lakes
d. Pneumonia d. Is acquired from environmental sources
3. Which of the following agars should be used for cultur- 10. True or False
ing Legionella? _____ Humoral immunity plays an important role in
a. CIN the defense against Legionella.
b. BCYE _____ Sputum specimens should be treated with sulfu-
c. SMAC ric acid before culturing.
d. XLD _____ Respiratory secretions may be held for 24 hours
4. What is the specimen of choice for isolating Legionella? at room temperature before culturing.
a. Stool _____ Specimens collected by bronchial alveolar lavage
b. Urine should be concentrated tenfold before culturing.
c. Cerebrospinal fluid (CSF) 11. Matching: Match the correct term with the appropriate
d. Respiratory secretions description.
5. Which of the following is acceptable for therapy? _____ legionnaires’ a. self-limiting, nonfatal
a. Fluoroquinolones _____ Pontiac fever respiratory infection
b. Penicillin _____ BCYE b. defective organelle
c. Cephalosporin _____ urine Ag test trafficking
d. Aminoglycosides _____ dot c. intracellular multiplication
6. Legionella injects proteins into the host cell by: _____ icm d. pneumonic illness
a. Dot/icm e. agar recommended for
b. Viral multiplication Legionella isolation
c. Damaging the epithelial cell lining the blood vessels f. rapid detection by EIA
d. Inhibiting the host defense
7. Legionella can be definitively diagnosed by a:
a. Twofold rise in anti-Legionella antibody with an IFA
b. Single serum with a titer of 128
c. Monoclonal immunofluorescent stain
d. Gram stain
Tille: Bailey & Scott’s Diagnostic Microbiology, 14th Edition
Chapter 34: Legionella

Answers to Case Studies and Review Questions

Case Studies

Case Study 34-1

1. The urinary antigen test effectively detects Legionella pneumophila type 1 but is less
often positive in infections with the other types of L. pneumophila.
2. Legionella is an opportunistic disease, meaning that an underlying factor in the host
generally accounts for the ability of the bacteria to cause infection. Risk factors include
lowered immune system from corticosteroid agents, malignancy, or age. Infants and older
adults naturally have a lowered immune system. Surprisingly, patients with acquired
immunodeficiency syndrome (AIDS) do not commonly develop infections with
Legionella, despite their lowered immune system. Other risk factors would include a
heavy exposure to water containing Legionella sp.
3. The diagnosis is hampered by the lack of sputum production by the patient to provide a
good specimen. Cultures are limited because of the long incubation time for growth, the
need for special media with a rather short outdate, the need to keep the cultures moist to
grow the bacteria, the need to inhibit overgrowth of other bacteria by using antibiotics in
the media or by acid treatment of the specimen, the inhibitory effect of yeast and of CO2
on the organism, and the lack of biochemical tests to identify the organism once cultured.
Nucleic acid amplification assays offer rapid results and increased sensitivity.
4. The body’s immune response to Legionella does not include the usual febrile response
with polymorphonuclear cells. Sputum, if produced at all, will be watery and lack an
increase in WBCs. A more invasive specimen is sometimes needed to make the
diagnosis.

Case Study 34-2

1. None are indicated. No remarkable history of exposure or chronic respiratory disease is


revealed. The only item that suggests a potential chronic respiratory disease would be the
history of smoking; however, no apparent damage is noted from the examination.
Laboratory results indicate a bacterial pneumonia, as evident by a chest x-ray image and
WBC neutrophilia.
2. Serologic studies for pandemic influenza H1N1 or a urine antigen test for Legionella
may be indicated. Possibly, an examination for Mycobacterium sp. may also be indicated.
3. Atypical pneumonias, especially with Legionella sp., often demonstrate a prodrome of
diarrhea and headache. Hyponatremia is a common feature of the disease. Most often,
cases are sporadic and not part of an outbreak. This patient had no history that would lead
the physician to consider an infection or a predisposition for Legionella infection.
Persistent hypoxia and severe fever despite treatment is common, along with a
nonpurulent cough.

Copyright © 2017, Elsevier Inc. All Rights Reserved.


Answers to Case Studies and Review Questions 34-2

Review Questions

Multiple Choice

1. b; 2. a; 3. b; 4. d; 5. a; 6. a; 7. c; 8. c; 9. b

True/False

F, F, F, T

Matching

d, a, e, f, b, c

Copyright © 2017, Elsevier Inc. All Rights Reserved.


CHAPTER 35 Brucella 473.e1

Chapter Review
1. Organisms belonging to the genus Brucella are: 8. When a specimen other than blood is tested, which
a. Motile agar is recommended?
b. Gram-positive bacilli a. Blood agar
c. Anaerobic b. Chocolate agar
d. Facultative intracellular c. Brucella agar
2. Humans become infected with Brucella spp. by all of d. MacConkey agar
the following means except: 9. Which test should be used for screening patients in an
a. Direct contact with infected animal parts outbreak of possible brucellosis?
b. Ingestion of unpasteurized milk a. ELISA
c. Person-to-person contact b. Laminate flow dipstick
d. Inhalation of infected aerosolized particles. c. Immunocapture agglutination
3. Which Brucella species may require CO2 for growth, is d. Coombs test
urease positive in 2 hours, and is inhibited by thiamine 10. True or False
dye? _____ Brucella spp. are capable of survival in soil for
a. B. abortus longer than 2 months.
b. B. melitensis _____ Brucella organisms tend to localize in tissue such
c. B. suis as placental tissue.
d. B. canis _____ B. canis and B. suis are the most virulent species
4. What titer in the SAT is considered diagnostic and cor- for humans.
relates to the clinical findings? _____ Isolation of Brucella organisms is a definitive
a. 1:1 diagnosis of brucellosis.
b. 1:4 _____ Most strains of B. abortus show equal growth in
c. 1:80 air and in a candle jar.
d. 1:160 11. Matching: Match each term with the appropriate
5. Why should the microbiology laboratory be notified description.
when brucellosis is suspected? _____ facultative a. infection with Brucella
a. It is pathogenic for humans. intracellular spp.
b. It is a vaccine-preventable disease. _____ category B select b. contains additive of
c. Brucella organisms are category B select agents. agent horse/rabbit serum
d. It is a zoonotic disease. _____ brucellosis c. rapid test for presump-
6. Which of the following Brucella spp. is not considered _____ Brucella agar tive Brucella infection
a bioterrorism agent? _____ particle d. transmitted primarily
a. B. abortus agglutination by aerosols
b. B. melitensis _____ unpasteurized e. most common means
c. B. suis animal milk of Brucella transmission
d. B. canis _____ apoptosis f. can exist in extracellu-
7. Resolution of a Brucella infection depends on what? lar and intracellular
a. Host’s nutritional and immune status environments
b. Size of inoculum g. programmed cell
c. Route of infection death
d. All of the above
Tille: Bailey & Scott’s Diagnostic Microbiology, 14th Edition
Chapter 35: Brucella

Answers to Case Studies and Review Questions

Case Studies

Case Study 35-1

1. Because airborne transmission of Brucella and Francisella has led to laboratory-


acquired infections, the cultures should be handled in a biological safety cabinet
whenever these genera are in the differential.
2. A positive rapid urease test will presumptively identify Brucella spp., one of the few
coccobacilli that are both oxidase- and catalase-positive and found in normally sterile site
specimens. Although several other coccobacilli are urease-positive, including Bordetella
spp. and Oligella ureolytica, they are motile, grow on MacConkey agar, or would be
unlikely in a joint specimen.
3. Pasteurization is not always used for cheese and milk products in Middle Eastern
countries. Brucella infection is chronic in nature and had probably been present in this
patient for some time. In 40% of untreated cases, the infection is present in the joints. The
foreign body in her knee makes her more susceptible to infections in that location.
4. The diagnosis can be confirmed by serologic testing for Brucella agglutinins. A titer of
1:160 is considered positive, but a negative titer does not rule out the disease. The
organism identification can also be confirmed with serologic reagents. Without
appropriate diagnosis and treatment, the organism will not be eradicated. Laboratory
susceptibility testing is unreliable, leading to erroneous results. If the identification is not
correct, then inappropriate treatment can result. Usually the local health department
confirms the identification. In fact, Brucella disease is reportable and on the list of
potential bioterrorist organisms.

Review Questions

Multiple Choice

1. d; 2. c; 3. a; 4. d; 5. c; 6. d; 7. d; 8. c; 9. b

True/False

T, T, F, T, F

Matching

f, d, a, b, c, e, g

Copyright © 2017, Elsevier Inc. All Rights Reserved.


CHAPTER 36 Bordetella pertussis, Bordetella parapertussis, and Related Species 479.e1

Chapter Review
1. The primary pathogen of whooping cough is: 9. An organism is cultured on Bordet-Gengou agar and
a. B. bronchiseptica grown at 35°C in a humid atmosphere. The organism is
b. B. holmesii detected at 72 hours. This organism is most likely:
c. B. pertussis a. B. pertussis
d. B. parapertussis b. B. parapertussis
2. What is the specimen of choice for culturing B. pertussis? c. B. holmesii
a. Throat d. More information is needed.
b. NP swab 10. Which of the following is not an effective transport
c. Sputum medium for Bordetella spp. when the specimen will be
d. Anterior nose sent overnight to a reference laboratory?
3. Which selective media should not be used for primary a. Cold casein hydrolysate
isolation of B. pertussis? b. Half-strength Regan-Lowe agar
a. Bordet-Gengou c. Fluid-transport medium
b. Modified Jones-Kendrick charcoal d. Dry swab in air for PCR testing
c. Regan-Lowe 11. True or False
d. BCYE _____ B. pertussis infections are endemic as a result of
4. Which specimen is not acceptable for B. pertussis PCR? waning vaccine-induced immunity.
a. NP rayon swab _____ B. pertussis organisms paralyze the cilia with tra-
b. NP Dacron swab cheal cytotoxins.
c. NP calcium-alginate swab _____ Bordetella organisms are detectable by culture
d. NP aspirate for up to 2 months after the start of paroxysms.
5. Pertussis cultures: _____ Whole-cell vaccines are now used instead of an
a. Can be considered negative after 3 days of incubation acellular vaccine because of adverse reactions
b. Should be incubated in CO2 seen with the latter.
c. Resemble mercury droplets _____ B. holmesii has been cultured in individuals
d. Should be incubated at room temperature working with poultry.
6. A PCR sample for B. pertussis tests positive for IS1001 12. Matching: Match each term with the appropriate
from an NP sample. The technologist should: description.
a. Repeat the test _____ Whooping a. decreasing severity of
b. Confirm with serology (ELISA-specific test) cough coughing spells
c. Report as positive _____ Catarrhal stage b. severe and violent cough
d. Collect a new specimen _____ Paroxysmal c. potato infusion agar
7. An NP sample for B. pertussis is DFA-positive. The tech- stage d. charcoal agar
nologist should: _____ Convalescent e. classic pertussis
a. Repeat the test stage f. pertussis toxin
b. Confirm with serology (ELISA-specific test) _____ Regan-Lowe g. mild cold, runny nose
c. Report as positive _____ Bordet-Gengou
d. Collect a new specimen _____ PT
8. The most reliable serologic test for the diagnosis of
B. pertussis in adults and adolescents is:
a. Complement fixation
b. Agglutination
c. Anti-PT ELISA
d. Enzyme immunoassay (EIA)
Tille: Bailey & Scott’s Diagnostic Microbiology, 14th Edition
Chapter 36: Bordetella pertussis and Bordetella parapertussis

Answers to Case Studies and Review Questions

Case Studies

Case Study 36-1

1. The patient had a child at home who had no antibodies to Bordetella pertussis.
Pertussis in the newborn is often fatal. The infectious disease (ID) service wanted to be
certain that the mother was not infectious to the baby. Treatment of the mother for 5 days
with erythromycin or a similar macrolide should protect the baby. Because the number of
pertussis cases in the United States is increasing, a vaccine was licensed in 2005 for use
in teenagers and adults. Hopefully, the use of the vaccine will decrease the number of
days lost in school and the workplace and decrease the number of infants exposed to B.
pertussis.
2. Some reasons include the following: Specimen collection is usually not adequate, the
organism is quite labile in transport, the culture typically takes 5 days for colonies to
appear, and special media must be used for cultivation. A PCR test for this bacterium has
increased the ability to rapidly detect B. pertussis.
3. The PCR assay is based on a transposon that is present in multiple copies in the
bacteria. Although this transposon is present in other species of Bordetella, the
combination of a positive result with symptoms of cough for longer than 2 weeks
provides sufficient information for the definitive diagnosis.
4. The organism is a tiny coccobacilli that is catalase- and oxidase-positive, nonmotile,
and urease-negative. It is unable to grow on MacConkey or blood agar, but it grows on
Regan-Lowe medium.

Review Questions

Multiple Choice

1. c; 2. b; 3. d; 4. c; 5. c; 6. c; 7. b; 8. c; 9. d; 10. d

True/False

T, T, F, F, F

Matching

e, g, b, a, d, c, f

Copyright © 2017, Elsevier Inc. All Rights Reserved.


CHAPTER 37 Francisella 483.e1

Chapter Review
1. Which of the following media should be chosen for 7. The typical clinical presentation of F. tularensis includes:
primary isolation of F. tularensis? a. A lesion that develops into an ulcer at the site of
a. Blood inoculation
b. Cystine-heart b. High temperature, chills, headache, and aching
c. MacConkey c. Enlargement and necrosis of lymph nodes
d. Chocolate d. All of the above
2. All of the following can cause humans to become in- 8. F. philomiragia biochemically differs from F. tularensis by:
fected with F. tularensis except: a. Nitrate and catalase reactions
a. Ticks b. Sodium hippurate and cephalothin reactions
b. Inhalation c. Oxidase and gelatin hydrolysis reactions
c. Deerflies d. Ornithine and indole reactions
d. Person-to-person transmission 9. True or False
3. Which statement about F. tularensis is true? _____ Most cases of F. tularensis occur sporadically
a. Tularemia can be handled on an open bench. during the fall months.
b. Tularemia is sexually transmitted. _____ Most F. tularensis cases occur in Wyoming,
c. Tularemia causes vomiting and diarrhea-like symptoms. Montana, and Utah.
d. Tularemia is one of the most common laboratory- _____ F. novicida and F. philomiragia are present in the
acquired infections. environment.
4. Optimal recovery of F. tularensis is acquired by: _____ F. tularensis infections can range from mild and
a. Increased CO2 self-limiting to fatal.
b. Enriched media containing cysteine _____ Serum antibody detection is useful for all forms
c. 24-hour growth at room temperature of tularemia.
d. Anaerobic conditions 10. Matching: Match each term with the correct description.
5. Which of the following may indicate the possibility of _____ Cysteine a. most severe form of
a Francisella species being identified? _____ Ulceroglandular tularemia
a. Poorly staining gram-negative rod. _____ Oculoglandular b. ulcer at point of
b. Oxidase-positive. _____ Pneumonic inoculation
c. Organism grows on MacConkey agar at 24 hours. tularemia c. requirement for
d. Organism satellites on the X and V test. _____ Systemic tularemia Francisella growth
6. The capsule of F. tularensis is necessary for the organism to: d. acute illness with
a. Resist Gram staining septicemia
b. Avoid immediate destruction by polymorphonuclear e. conjunctivitis
neutrophils
c. Infect humans with an infection dose of less than five
colony-forming units
d. Replicate in the lungs
Tille: Bailey & Scott’s Diagnostic Microbiology, 14th Edition
Chapter 37: Francisella

Answers to Case Studies and Review Questions

Case Studies

Case Study 37-1

1. The most commonly misidentified microorganism submitted as Francisella tularensis


is Haemophilus influenzae. Microbiologists do not expect F. tularensis to grow on
chocolate agar, but today the commercial vendors make a very high-quality medium that
allows this fastidious organism to grow. Lack of growth on sheep blood agar, and lack of
satellite colonies around a Staphylococcus spp. streak or dot, which supplies
nicotinamide-adenine-dinucleotide (NAD) to the organism, rules out Haemophilus.
2. The culture should be handled with Biosafety Level 3 precautions, which are the same
precautions used to handle Mycobacterium tuberculosis. However, it is recommended
that all culture manipulations should be carried out in a specialized laboratory and should
be sent to the state health department. The physician should be notified of the possible
identification of F. tularensis. In addition, the local epidemiologist should be immediately
contacted if a bioterrorist event associated with the isolation of the organism cannot be
ruled out.
3. This test is part of the recommended protocol for the identification of F. tularensis for
sentinel laboratories. F. tularensis has a β-lactamase that is active against all penicillins
and cephalosporins, including imipenem. A positive test alerts the microbiologist that this
is not a fastidious HACEK (AACEK) bacterium (Chapter 30) and that it does not grow
on blood agar. HACEK (AACEK) bacteria are usually β-lactamase–negative. Because
performing susceptibility testing on F. tularensis is difficult, the positive β-lactamase test
can be a guide to therapy.

Review Questions

Multiple Choice

1. b; 2. d; 3. d; 4. b; 5. a; 6. b; 7. d; 8. c

True/False

F, F, T, T, T

Matching

c, b, e, a, d

Copyright © 2017, Elsevier Inc. All Rights Reserved.


CHAPTER 38 Streptobacillus moniliformis and Spirillum minus 487.e1

Chapter Review
1. When S. moniliformis is acquired by ingestion, the disease 6. The natural habitat of S. moniliformis is the upper respi-
is called: ratory tract of:
a. Trench fever a. African dwarf frogs
b. Haverhill fever b. Wild and laboratory rats
c. Cat-scratch disease c. Prairie rattlesnakes
d. Lyme disease d. Black-footed ferrets
2. Which of the following is not required for growth of 7. S. moniliformis and S. minus can be differentiated symp-
S. moniliformis? tomatically by:
a. Blood a. Febrile episodes
b. Ascitic fluid b. Rash
c. Cystine c. Arthritis
d. Serum d. Headache
3. All of the following have been used to describe colonies 8. All of the following are necessary for the growth of
of S. moniliformis except: S. moniliformis except:
a. Fluff balls a. The presence of blood, ascitic fluid, or serum
b. Mercury drops b. 48 hours of incubation at 37°C
c. Bread crumbs c. 5% to 10% CO2 environment
d. Fried egg d. MacConkey agar in ambient air
4. S. minus also causes rat-bite fever in humans and is 9. On Giemsa staining, S. minus appears as:
referred to as: a. Club-shaped cells
a. Haverhill fever b. Extreme pleomorphism
b. Legionnaire’s disease c. Spiral organisms with two or three coils of polytri-
c. Hantavirus chous polar flagella
d. Sodoku d. Bipolar-staining coccoid forms
5. Definitive diagnosis of S. minus is made by:
a. Mouse or guinea pig inoculation
b. Serologic tests
c. Culture of lymph node tissue
d. Gram stain
Tille: Bailey & Scott’s Diagnostic Microbiology, 14th Edition
Chapter 38: Streptobacillus moniliformis and Spirillum minus

Answers to Case Studies and Review Questions

Case Studies

Case Study 38-1

1. Streptobacillus moniliformis is susceptible to sodium polyanethol sulfonate (SPS), the


anticoagulant used in blood culture bottles. By adding more blood, all of the SPS would
be bound up, and the inhibitory effect of the anticoagulant would be minimized. This was
successful and the organisms grew. The effort to grow it from the tube failed because too
much SPS was in the tube used to draw the blood.
2. S. moniliformis will not grow on laboratory media unless sufficient blood, ascitic fluid,
or horse serum is in the medium. Some strains require 8% to 10% CO2. In the case of this
patient, tiny colonies finally appeared on the blood agar plate after 72 hours in 8% CO2.
3. Once colonies are apparent, identification is based on negative results for oxidase,
catalase, nitrate, and indole. One must be certain that growth is in the nitrate tube; adding
serum to the broth may be necessary. In addition, the Gram stain is critical to the
identification. L-form colonies or long rods with thick knobs are characteristic (Figure
38-1), whereas coccoid forms or rods with tapered ends are more likely Capnocytophaga.
Another genus, Leptotrichia, is similar, but it does not usually grow on laboratory media.
In addition to biochemical testing, communicating with the physician is important to
ensure that the identification fits with the clinical picture.

Review Questions

Multiple Choice

1. b; 2. c; 3. b; 4. d; 5. a; 6. b; 7. c; 8. d; 9. c

Copyright © 2017, Elsevier Inc. All Rights Reserved.


CHAPTER 39 Neisseria and Moraxella catarrhalis 497.e1

Chapter Review
1. Which organism is not a normal inhabitant of the 9. An organism grows on blood agar at room temperature
respiratory tract? and nutrient agar at 37°C. The organism has the ability
a. M. catarrhalis to utilize carbohydrates and is nitrate-positive, DNase-
b. N. meningitidis positive, and ONPG-negative. This organism is:
c. N. mucosa a. N. gonorrhoeae
d. N. gonorrhoeae b. N. meningitidis
2. Which of the following is a leading cause of fatal bacte- c. N. lactamica
rial meningitis? d. M. catarrhalis
a. B. pertussis 10. All of the following are advantages of using amplified
b. M. catarrhalis testing over nonamplified testing except:
c. N. meningitidis a. It is more sensitive.
d. N. gonorrhoeae b. It is suitable for large screening programs.
3. Which of the following is a leading sexually transmit- c. It is admissible in medicolegal cases.
ted infection? d. It allows testing for Chlamydia trachomatis at the
a. N. gonorrhoeae same time.
b. N. meningitidis 11. True or False
c. N. sicca _____ Men who have sex with men (MSM) are the
d. N. mucosa primary reservoirs for the dissemination of
4. Sensitivity testing for N. gonorrhoeae can be accom- N. gonorrhoeae in the human population.
plished by: _____ CTA sugars do not require subculture for
a. Enzyme immunoassay (EIA) growth and can be read in 4 hours.
b. Gram stain _____ Infection with M. catarrhalis may lead to severe
c. Fluorescent antibody stain disseminated disease.
d. Amplified assays _____ Genital isolates of Neisseria spp. in adults with
5. Which of the following media is not acceptable for high-risk behavior must be completely identified.
primary growth of N. gonorrhoeae? _____ The presence of prolyl-hydroxylprolyl aminopepti-
a. Chocolate dase alone identifies an organism as N. meningitidis.
b. Thayer-Martin 12. Matching: Match the correct term with the appropriate
c. Blood description.
d. Martin-Lewis _____ N. gonorrhoeae a. nonpathogenic
6. Pathogenic M. catarrhalis grows best under which _____ N. meningitidis b. CO2-generating system
conditions? _____ Thayer-Martin c. N. meningitidis groups
a. Anaerobic atmosphere _____ Saprophytic seen in the United States
b. 42°C _____ JEMBEC d. Antigens in N. meningiti-
c. Increased CO2 _____ A, B, C, Y, and dis vaccine
d. 25°C W135 e. Leading cause of STIs
7. N. gonorrhoeae can be differentiated from N. meningiti- _____ A, C, Y, and f. Selective agar for cultur-
dis by: W135 ing Neisseria spp.
a. Dextrose fermentation g. Cause of fatal bacterial
b. Maltose fermentation meningitis
c. Lactose fermentation 13. Short Answer
d. Nitrate reduction A synovial fluid sample is submitted for culture during the
8. All of the following statements about N. gonorrhoeae night shift at approximately 2 a.m. The laboratory staff
are true except: is extremely busy with tests and supplying blood prod-
a. Particle agglutination methods are available for ucts for victims of a serious car accident. The synovial
identification. fluid sample is placed in the refrigerator until sufficient
b. The sample should be incubated at 36°C for 72 hours time is available to plate the sample to the appropriate
in a CO2-enriched environment. media. At approximately 6 a.m., the day shift person-
c. Penicillin is the therapy of choice. nel note the specimen in the refrigerator. Should the
d. The organism may appear to have up to five differ- sample be cultured after refrigeration if the physician
ent colony types. suspects an isolate of the organisms discussed in this
chapter? How should the laboratory staff proceed?
Should the specimen be processed? Explain.
Tille: Bailey & Scott’s Diagnostic Microbiology, 14th Edition
Chapter 39: Neisseria and Moraxella catarrhalis

Answers to Case Studies and Review Questions

Case Studies

Case Study 39-1

1. Moraxella catarrhalis, non-typeable Haemophilus influenzae (Chapter 31) and


Streptococcus pneumoniae (Chapter 14).
2. M. catarrhalis is negative for β-galactosidase, γ-glutamylaminopeptidase and prolyl-
hydroxylprolyl aminopeptidase. It is also capable of growth on blood agar at 22C and
nutrient ager at 35C, reduces nitrate to nitrite, negative in carbohydrate utilization tests,
and positive for DNase.
3. Although the isolate was susceptible to penicillin, additional factors affect the
virulence of the organism (capsular subtype) and host factors including age (depressed
immune system) and comorbidities such as debilitating disease; in this case COPD.

Review Questions

Multiple Choice

1. d; 2. c; 3. a; 4. d; 5. c; 6. c; 7. b; 8. c; 9. d; 10. c

True/False

F, F, F, F, F

Matching

e, g, f, a, b, c, d

Short Answer

1. The synovial fluid should not be tested for Neisseria spp. Neisseria spp. are extremely
sensitive to cold temperatures. They should always be transported to the laboratory under
ambient conditions and tested immediately. If they cannot be tested within 6 hours, they
should be held at room temperature with enhanced CO2 for optimal recovery. The staff
should not test this specimen for Neisseria spp. and should inform the physician.

Copyright © 2017, Elsevier Inc. All Rights Reserved.


CHAPTER 40 Overview and General Laboratory Considerations 512.e1

Chapter Review
1. All of the following specimens are acceptable for anaero- 6. A gram-negative rod is isolated from a clinical specimen
bic culture except: collected from a female patient with a genital swab. The
a. Bile organism is characterized as kanamycin-sensitive, colistin-
b. Joint fluid sensitive, vancomycin-resistant, bile hydrolysis–negative,
c. Skin swab and positive for urea hydrolysis and nitrate reduction. This
d. Aspirate organism is most likely:
2. After preliminary plating, all isolated colonies should be a. Fusobacterium spp.
subcultured and incubated on: b. B. ureolyticus
a. BA in 2% O2 c. Veillonella spp.
b. CHOC in 5% CO2 d. C. difficile
c. CHOC in 0% O2 7. Matching: Match each term with the associated term.
d. BA in 0% O2 and 5% CO2 _____ Mobiluncus spp. a. pseudomembrane
3. Specimen characteristics of an anaerobic infection include: _____ Actinomyces b. gas gangrene
a. Foul odor, presence of sulfur granules, and green fluo- _____ Bifidobacterium c. molar tooth
rescence _____ C. perfringens d. lock jaw
b. Foul odor, presence of metachromatic granules, and _____ Bacteroides spp. e. bacterial vaginosis
green fluorescence _____ C. difficile f. dog bones
c. Foul odor, sulfur smell, and red fluorescence _____ C. septicum g. fried egg
d. Foul odor, presence of sulfur granules, and red fluo- _____ Eubacterium h. seagull wing
rescence _____ C. tetani i. esculin hydrolysis
4. Which primary plating medium consists of a Brucella _____ B. thetaiotaomicron j. Medusa head
agar base containing kanamycin and vancomycin? _____ F. varium k. punctiform
a. BBE 8. Short Answer
b. PEA Anaerobic cultures are incubated in a GasPak chamber.
c. EYA Upon opening and examining the culture plates, the
d. LKV microbiologist notes that, according to the indicator
5. A wound specimen is collected on a swab and placed in result, the anaerobic jar did not operate properly.
transport media. The swab is placed in the refrigerator What may have caused the indicator to fail to func-
for approximately 30 minutes before transport to the tion properly? What is the next step in the laboratory
laboratory. The laboratory technologist immediately with regard to the incubated cultures and isolates?
plates the swab to BBE, LKV, PEA, CHOC, and BA and
inoculates a THIO tube. All plates and the tube are incu-
bated anaerobically for 7 days without growth. Which of
the following is the likely reason that no anaerobic bacte-
ria were cultivated?
a. Poor transport method
b. Excessive exposure to air during processing
c. Inadequate media
d. None of the above
Tille: Bailey & Scott’s Diagnostic Microbiology, 14th Edition
Chapter 40: Overview and General Considerations

Answers to Case Studies and Review Questions

Review Questions

Multiple Choice

1. c; 2. d; 3. d; 4. d; 5. d; 6. b

Matching

e, c, f, b, i, a, j, h, d, k, g

Short Answer

1. The system requires the addition of water and no water was added to the GasPak
system, no anaerobic environment would be generated, and jar may have not been
properly sealed. The cultures would not be viable, requiring recollection of specimens for
new cultures.

Copyright © 2017, Elsevier Inc. All Rights Reserved.


CHAPTER 41 Overview of Anaerobic Organisms 523.e1

Chapter Review
1. A rectal swab is submitted for anaerobic culture. The 8. A patient presents to the emergency department com-
laboratory scientist should: plaining of a foul-smelling, purulent wound on his hand.
a. Set up the swab on the routine laboratory media for The patient states that he injured his hand 3 days earlier
anaerobic culture while working in the yard. Microbiology cultures are or-
b. Set up the swab on routine stool and anaerobic media dered, and after 72 hours, the following results are ob-
c. Reject the specimen tained: gram-negative rod, growth on anaerobic blood
d. Use the swab for toxin testing to identify C. difficile agar, positive aerotolerance, weakly catalase-positive,
2. Sulfur granules in a clinical specimen indicate the colistin-resistant, kanamycin-resistant, and vancomycin-
presence of: sensitive. The organism is most likely:
a. Clostridium spp. a. B. fragilis
b. Fusobacterium spp. b. C. perfringens
c. Actinomyces spp. c. F. nucleatum
d. Peptostreptococcus spp. d. B. thetaiotaomicron
3. The ethanol shock procedure is used to differentiate: 9. Matching: Match each term with the best description.
a. Actinomyces and Bifidobacterium spp.
_____ Mobiluncus spp. a. sulfur granules
b. Prevotella and Porphyromonas spp.
_____ C. botulinum b. propionic acid
c. Clostridium and Bacteroides spp.
_____ C. difficile c. brick red fluorescence
d. Bacteroides and Actinomyces spp.
_____ Propionibacterium d. pseudomembranous
4. The physician suspects a case of C. perfringens–associated
spp. colitis
enteritis necroticans. The specimen(s) required for diag-
_____ P. micra e. pointed- and blunt-
nosis is/are:
_____ Actinomyces ended cell morphology
a. Three blood cultures and a stool specimen
_____ Prevotella spp. f. infant botulism
b. A stool specimen
_____ Leptotrichia spp. g. flaccid paralysis
c. A colonic tissue biopsy
_____ C. perfringens h. bacterial vaginosis
d. Stool and a colonic tissue biopsy
_____ honey i. bile resistant
5. In the diagnosis of C. difficile infection (CDI), all of the
_____ bioassay j. neutropenic
following are true except:
_____ Bacteroides spp. enterocolitis
a. Specimens may be stored for up to 3 days before
k. BoNT
processing.
l. SPS sensitive
b. Specimen collection requires a fresh stool for culture 10. Short Answer
and toxicity testing. (1) Why is it necessary to Gram stain the clinical speci-
c. Liquid stool is used primarily to provide material men upon receipt in the laboratory when an anaerobic
for ease of toxicity testing. culture is ordered?
d. Liquid or partially formed stool is required to pre- (2) An organism is identified that is resistant to all
vent false diagnosis of a carrier state. three special potency disks (RRR) and demon-
6. A suprapubic urine aspirate is collected and submitted strates bile resistance. What is the likely identity of
for anaerobic culture. After 72 hours of incubation and the organism?
biochemical testing, the organism is identified as a (3) After testing with the special potency disks, you
gram-positive, aerotolerant, catalase-negative rod. The identify an organism as a gram-positive cocci. The
organism is most likely: Gram stain indicates that the organism is gram
a. Lactobacillus sp. negative. What is the reason (or reasons) for the
b. Bifidobacterium sp. discrepancy? What would be the next step in labo-
c. Eubacterium sp. ratory diagnosis?
d. C. septicum
7. Transmission of anaerobic infections is predominantly
a result of:
a. Medical device inoculation
b. Nosocomial spread
c. Contaminated food and water
d. Normal microbiota
Tille: Bailey & Scott’s Diagnostic Microbiology, 14th Edition
Chapter 41: Laboratory Considerations

Answers to Case Studies and Review Questions

Review Questions

Multiple Choice

1. c; 2. c; 3. c; 4. a; 5. c; 6. a; 7. d; 8. b

Matching

h, g, d, b, l, a, c, e, j, f, k, i

Short Answer

1. The Gram-stain reaction can provide the laboratory personnel with an indication as to
what the follow-up testing and identification procedure will be for the anaerobic
organism.
2. Bacteroides fragilis.
3. Keeping in mind the variability associated with the isolation of anaerobic bacteria, a
number of possibilities exist: (1) The organisms were exposed to oxygen and are dead,
resulting in the breakdown of the bacterial cell wall and falsely negative staining pink or
gram-negative. (2) The organism demonstrates gram-variability, which must be
considered when interpreting the characteristics in combination with biochemical
identification. (3) Some anaerobes tend to be pleomorphic and gram-variable, such as in
Clostridium perfringens, which can appear as rods or coccoid. The laboratory scientist
should review all biochemical tests completed before the potency disks and determine
whether the organism was properly processed. After the review, the laboratory
professional may wish to request a secondary specimen and should fully explain the
results to the physician, allowing the physician to determine proper treatment and follow-
up.

Copyright © 2017, Elsevier Inc. All Rights Reserved.


CHAPTER 42 Mycobacteria 552.e1

Chapter Review
1. Photochromogens are capable of pigment production 8. A clinical Mycobacterium isolate is grown in the dark
when: for approximately 21 days. After exposure to light for
a. Exposed to light for approximately 1 hour 8 hours, the organism produces a yellow pigment. The
b. Grown in the presence of CO2 organism is most likely:
c. Incubated in ultraviolet radiation a. M. tuberculosis
d. Incubated in the dark at 30°C, followed by brief b. M. fortuitum
exposure to regular light c. M. bovis
2. The medium most often used for isolation and cultivation d. M. kansasii
of Mycobacterium spp. is: 9. A urine culture produces a few colonies of acid-fast ba-
a. Cystine blood agar cilli after approximately 14 days. The colonies are rough
b. Loeffler’s medium and nonpigmented. The niacin test is weakly positive,
c. Sheep blood agar and the nitrate test is positive. The most appropriate
d. Löwenstein-Jensen medium action for the technologist considering the potential
3. In patients who are HIV positive or who have AIDS, isolate is:
chronic pulmonary disease may be associated with in- a. Report the organism as M. tuberculosis so that the
fection as a result of: physician may initiate therapy.
a. M. gordonae b. Set up a sensitivity test to determine whether the
b. M. kansasii patient requires treatment or whether the organism
c. M. avium complex is simply a contaminant.
d. M. fortuitum c. Repeat the niacin test and report presumptive
4. The disease-producing capability of M. tuberculosis is identification of M. tuberculosis if the test appears
associated with the ability to: positive.
a. Produce a massive burst of exotoxin, destroying d. Subculture the isolate and carry out a series of biochem-
lung tissue ical tests, including niacin, nitrate, and Tween 80, and
b. Escape death by multiplication in alveolar macro- notify the physician that no results are available at this
phages time.
c. Produce a complex endotoxin more deadly than 10. The unique chemical structure of the cell wall of Myco-
gram-negative organisms bacterium spp. is associated with the presence of:
d. Trick the humoral immune response in the host a. N-glycolylmuramic acid and a decrease in lipid content
5. For the recovery of Mycobacterium spp., which of the b. N-acetylmuramic acid and a decrease in lipid content
following is useful as a mucolytic, alkaline reagent that c. N-glycolylmuramic acid and an increase in lipid
digests and decontaminates a sputum sample? content
a. Zephiran-trisodium phosphate d. N-acetylmuramic acid and an increase in lipid content
b. N-acetyl-L-cystine 11. A patient suspected of having an M. tuberculosis infec-
c. NaOH and oxalic acid tion is admitted to the hospital at 7 a.m. A sputum
d. N-acetyl-L-cystine and NaOH specimen is collected at that time. The acid-fast stain
6. Which of the following tests is most definitive for the reveals acid-fast bacilli. Two days later, a specimen
identification of M. tuberculosis? collected after lunch reveals no acid-fast bacilli. The
a. Catalase production laboratory technologist should:
b. Tween 80 hydrolysis a. Reexamine the initial specimen and notify the phy-
c. Production of niacin sician of the laboratory error.
d. Reduction of nitrate b. Report the second stain as negative for the presence
7. An acid-fast bacillus has been isolated from sputum in a of acid-fast bacilli.
patient suspected of having active pulmonary tubercu- c. Reject the second specimen and request that a new
losis. After incubation in the dark and exposure to light, specimen be collected immediately.
the isolated organism appears to be a slow-growing, d. Report the results of the second specimen, noting the
cream to tan colony. An SQ catalase reaction of 42 mm error in the collection, and request a new specimen.
is noted. The organism is most likely:
a. M. kansasii
b. M. fortuitum
c. M. avium complex
d. M. tuberculosis
552.e2 PA RT I I I Bacteriology

12. True/False 14. Short Answer


____ All Mycobacterium spp. demonstrate a strong (1) Describe the limitations and reasons for rejection
acid-fast staining reaction during all phases of of a clinical specimen for the recovery of Mycobac-
growth. terium species.
____ The term “complex” is used when species distinc- (2) Explain special considerations for processing gas-
tion is complicated and of no medical consequence. tric lavages, urine, and fecal specimens.
____ A blood culture that is positive for MAC is (3) For the isolation of Mycobacterium spp., what spec-
considered likely to be contaminated and should imens do not require decontamination? Explain.
be reported as such. (4) Describe the limitations associated with the identi-
____ Screening of specimens using a fluorescent stain fication and speciation of Mycobacterium isolates
may reduce the laboratory time required to diag- using molecular methods, including PCR and
nose tuberculosis and may improve detection of sequencing procedures.
positive smears. (5) Describe the recommended treatment for M. tu-
13. Matching: Match each term with the appropriate phrase berculosis and concerns associated with multiple-
or description. drug resistant isolates.
_____ M. leprae a. contaminated milk
_____ NTM b. positive PPD
_____ M. bovis (tuberculin skin test)
_____ Primary c. noncultivatable NTM
tuberculosis d. removes gram-negative
_____ Latent contaminating
tuberculosis organisms
_____ Disseminated e. no person-to-person
tuberculosis transmission noted
_____ Reactivation f. lacks signs and
tuberculosis symptoms
_____ Oxalic acid g. beaded or neutral
_____ Gram stain staining of organisms
h. result of suppressed
cellular immunity
i. pulmonary disease
Tille: Bailey & Scott’s Diagnostic Microbiology, 14th Edition
Chapter 42: Mycobacteria

Answers to Case Studies and Review Questions

Case Studies

Case Study 42-1

1. In meningitis caused by Mycobacterium tuberculosis, few organisms are detected.


Even with concentration of the CSF, smears can only detect 104 bacteria/mL. In addition,
smears are difficult because the mycobacteria are lipophilic and do not easily stick to the
slides. Cultures demonstrate a better recovery rate for mycobacteria. It is recommended
that 10 mL of liquid specimen be inoculated into a broth culture. PCR has the advantage
in that it can detect lower numbers of microorganisms and even nonviable
microorganisms from concentrated specimens.
2. M. tuberculosis is identified as the only species with a positive niacin test, provided
that the colony is buff colored, slow growing, and rough. In addition, the U.S. Food and
Drug Administration (FDA) has approved probe assays that are specific for the M.
tuberculosis complex. These tests are more rapid than the niacin test and can be
performed before colonies are present on solid media.
3. This complex includes M. tuberculosis, Mycobacterium africanum, and
Mycobacterium bovis, including M. bovis bacillus Calmette-Guérin (BCG). Other
members of the complex exist, but these are rarely seen in humans.
4. The laboratory can determine whether any other positive specimens were processed on
the same day as the patient in question. If so, then all of the positive isolates from that
day should be sent for DNA fingerprinting, PCR amplification, or sequencing. It is
unlikely that two strains will have the same fingerprint. If they do, then one specimen
may have possibly contaminated the other.

Review Questions

Multiple Choice

1. a; 2. d; 3. c; 4. b; 5. d; 6. c; 7. c; 8. d; 9. c; 10. c; 11. d

True/False

F, T, F, T

Matching

c, e, a, i, f, b, h, d, g

Short Answer

Copyright © 2017, Elsevier Inc. All Rights Reserved.


Answers to Case Studies and Review Questions 42-2

1. Specimens should be rejected according to the following guidelines: (1) insufficient


volume, (2) contamination with saliva, (3) dried swabs, (4) pooled sputum or urine, (5)
container has been compromised, broken, or leaking, (6) length of time from collection to
processing is too long.
2. Gastric lavages should be processed within 4 hours of collection or neutralized with
10% sodium carbonate (check with pH paper to determine that the specimen is at neutral
pH) and refrigerated until processed as for sputum. If more than 10 mL of watery-
appearing aspirate was obtained, the specimen can be centrifuged at 3600× g for 30
minutes, the supernatant decanted, and the sediment processed as for sputum.
Urine specimens should be divided into a maximum of four 50-mL centrifuge tubes and
centrifuged at 3600× g for 30 minutes. The supernatant should be decanted, leaving
approximately 2 mL of sediment in each tube. The tubes are vortexed to suspend the
sediments, and sediments are combined. If necessary, distilled water can be added to a
total volume of 10 mL. This urine concentrate is then treated as for sputum or with the
sputolysin-oxalic acid method.
For fecal specimens, approximately 0.2 g of stool (a portion about the size of a pea) is
emulsified in 11 mL of sterile, filtered, distilled water. The suspension is vortexed
thoroughly, and particulate matter is allowed to settle for 15 minutes. Ten milliliters of
the supernatant is then transferred to a 50-mL conical centrifuge tube and decontaminated
using the oxalic acid or NALC-NaOH method.
3. Tissues or body fluids collected aseptically usually do not require the digestion and
decontamination methods used with contaminated specimens.
4. PCR and sequencing rely on the comparison to databases. Problems remain in that the
sequences in some databases are not accurate, there is no present consensus as to the
quantitative definition of a genus or species based on 16S rRNA gene sequence data, and
procedures are not standardized. With improvements in Next Generation Sequencing and
the availability of molecular techniques, this is improving with more data collection.
5. Multidrug regimens are designed to prevent the selection of resistant mutants, therefore
requiring the treatment of tuberculosis in a combination of two or three drugs.

Copyright © 2017, Elsevier Inc. All Rights Reserved.


CHAPTER 43 Obligate Intracellular and Nonculturable Bacterial Agents 567.e1

Chapter Review
1. Which organism causes Rocky Mountain spotted fever? 8. True or False
a. Chlamydia trachomatis _____ The phase II form (small-cell variant) of C. bur-
b. Ehrlichia chaffeensis netii is like a spore and is not infectious.
c. Rickettsia rickettsii _____ A woman presents to her nurse practitioner com-
d. Coxiella burnetii plaining of a vaginal discharge. The nurse practitioner
2. Which serovar of C. trachomatis causes lymphogranuloma suspects C. trachomatis and collects a specimen in the
venereum? following manner: A large swab was used to remove
a. A all secretions from the cervix and then placed into the
b. C swab sheath and sent to the laboratory. This specimen
c. H was collected appropriately for C. trachomatis.
d. L1 9. Matching:
3. Which of the following organisms is acquired via exposure 1. Match each term with the appropriate definition.
to infected birds? _____ bubo a. bacteria found in a
a. Coxiella burnetii _____ proctitis lesion that are in the
b. Chlamydia psittaci _____ bartholinitis cytoplasm of mono-
c. Anaplasma phagocytophilum _____ salpingitis nuclear phagocytes or
d. Tropheryma whipplei _____ morulae histiocytes
4. A 24-year-old man presented to his physician in India _____ Donovan body b. infection and
because of a painless ulcer on his penis. Upon examina- inflammation of the
tion, the ulcer was erythematous, large, and “beefy.” fallopian tubes
When the lesion was sampled with a swab, it readily c. inflammation of the
started bleeding. In addition, inguinal lymph nodes were anus and rectum
swollen. Wright-Giemsa stain of material from the lesion d. swollen lymph node
revealed the presence of blue rods within the mononu- e. infection of the Bar-
clear cells that were stained more prominently on the tholin gland or duct
ends than in the middle. What organism is most likely f. intraphagosomal ag-
causing this lesion? gregates of replicating
a. Klebsiella granulomatis Ehrlichia
b. Chlamydia trachomatis 2. Match each disease with its causative agent (more than
c. Ehrlichia chaffeensis one may apply).
d. Tropheryma whipplei _____ trachoma a. Rickettsia rickettsii
5. A physician calls the laboratory and asks the medical labo- _____ scrub typhus b. Ehrlichia chaffeensis
ratory scientist which test to order to rule out Chlamydia _____ human monocytic c. Chlamydia
pneumoniae as a cause of pneumonia in a 17-year-old ehrlichiosis trachomatis
patient. What does the medical laboratory scientist tell _____ pneumonia d. Orientia
the physician? _____ lymphogranuloma tsutsugamushi
a. Sputum culture for routine bacterial pathogens venereum e. Chlamydia psittaci
b. Wright-Giemsa staining of bronchoalveolar lavage fluid _____ psittacosis f. Coxiella burnetii
c. Real-time PCR on a nasopharyngeal aspirate _____ granuloma g. Klebsiella
d. Serum testing for agglutination with Proteus OX-K, inguinale granulomatis
OX-19, and OX-2 _____ human granulocytic h. Rickettsia prowazekii
6. Transmission of O. tsutsugamushi is associated with what anaplasmosis i. Chlamydia
vector? _____ Q fever pneumoniae
a. Ticks _____ Whipple disease j. Tropheryma whipplei
b. Fleas k. Anaplasma
c. Lice phagocytophilum
d. Chiggers l. Rickettsia typhi
7. Which triad of symptoms is associated with rickettsial 3. Match each vector with the organism it transmits.
infections? _____ Lone star tick a. Anaplasma
a. Urethral discharge, fever, and dysuria (Amblyomma phagocytophilum
b. Coughing, production of sputum, and chest pain americanum) b. Orientia
c. Fever, headache, and rash _____ flea tsutsugamushi
d. Genital lesion, swollen inguinal lymph nodes, and _____ tick (Ixodes spp.) c. Rickettsia rickettsii
headache _____ chigger d. Rickettsia
_____ louse prowazekii
_____ tick (Dermacentor e. Rickettsia typhi
spp.) f. Ehrlichia chaffeensis
Tille: Bailey & Scott’s Diagnostic Microbiology, 14th Edition
Chapter 43: Obligate Intracellular and Nonculturable Bacterial Agents

Answers to Case Studies and Review Questions

Case Studies

Case Study 43-1

1. The patient has Ehrlichia chaffeensis, which was detected in the bone marrow smear
by the presence of morulae (i.e., variable-sized basophilic inclusions).
2. A zoonosis is a disease that humans contract from an association with animals or their
excretions. Humans are usually the accidental host in such diseases. In this case, the
vector of the disease is the tick and the usual host is the horse.
3. If the morula is not visualized in the peripheral smear, then the PCR and serologic
testing are currently the available methods to make the diagnosis. In this patient,
serologic testing was performed, and a single titer of 1:512 was obtained. Culture is
difficult in that these organisms have an intracellular life cycle that is limited to cell lines
that are difficult to grow in vitro.

Case Study 43-2

1. Nucleic acid amplification tests (NAAT) are used almost exclusively for genital tract
infections of chlamydia.
2. Doxycycline is recommended for treatment of chlamydia (2010 STD Treatment
Guidelines of the CDC). Metronidazole (Flagyl) is appropriate for parasite infection, such
as Trichomonas vaginalis. Since no Trichomonas were identified and chlamydia is the
diagnosis, metronidazole can be dropped from the patient’s treatment plan.
3. It causes significant infection and disease worldwide. In the United States, it is the
most common sexually transmitted bacterial pathogen and a major cause of pelvic
inflammatory disease (PID), ectopic pregnancy, and infertility. Approximately, 3 million
cases of Chlamydia trachomatis infection occurs annually in the United States.

Case Study 43-3

1. The etiologic agent is the gram-negative coccobacillus Rickettsia rickettsii transmitted


by bite of an infected tick of the genus Dermacentor.
2. The classic triad of symptoms common to many rickettsial diseases, including RMSF,
is fever, rash, and headache.
3. Laboratory methods for diagnosis of RMSF are serology, immunohistology, and PCR
with sequencing.

Review Questions

Multiple Choice

Copyright © 2017, Elsevier Inc. All Rights Reserved.


Answers to Case Studies and Review Questions 43-2

1. c; 2. d; 3. b; 4. a; 5. c; 6. d; 7. c

True/False

T, F

Matching

1. d, c, e, b, f, a
2. c, d, b, c and e and i, c, e, g, k, f, j
3. f, e, a, b, d, c

Copyright © 2017, Elsevier Inc. All Rights Reserved.


CHAPTER 44 Cell Wall-Deficient Bacteria: Mycoplasma and Ureaplasma 577.e1

Chapter Review
1. Why are Mycoplasmataceae difficult to cultivate in the 7. A patient presented to his doctor around 1 p.m., com-
clinical laboratory? plaining of frequent urination and abdominal pain. A
a. They lack cell walls. urine sample was collected by the patient at 8 a.m. and
b. They require serum components including sterols in submitted for culture. The routine urinalysis indicated a
the growth medium. positive leukocyte esterase test. All other tests were normal.
c. They are extremely sensitive to pH changes in the PCR was negative for Ureaplasma spp. After 24 hours of
environment. incubation, routine urine culture demonstrated the growth
d. All of the above of three different organisms with no significant pathogens
2. Mycoplasmas generally may be transmitted by all of the noted. What is the most likely cause for the conflicting
following except: laboratory results?
a. Direct sexual contact a. The original urine sample was not inoculated onto
b. Vertical transmission from mother to fetus proper growth media.
c. Oral sexual practices b. The PCR test was likely contaminated.
d. A contaminated toothbrush c. The sample delay in processing of the urine affected
3. All of the following are considered commensal organisms the viability of the organism.
capable of causing opportunistic infections except: d. Both A and C
a. U. urealyticum e. All of the above
b. M. fermentans 8. True or False
c. M. genitalium _____ All Mycoplasma spp. are capable of immune eva-
d. M. hominis sion through intracellular growth cycles.
e. M. pneumoniae _____ SPS in commercial blood culture media inhibits
f. U. parvum the growth of Mycoplasma spp.
4. Which of the following would be the most reliable sero- _____ Specimens collected for the isolation of Myco-
logic diagnosis for M. pneumoniae, considering the patho- plasma or Ureaplasma should be placed in trans-
genesis and the immunologic response and based on the port media and incubated at 37°C until processed.
descriptions provided? _____ Current CLSI standards recommend broth or
a. A single acute IgM titer agar dilution methods for susceptibility testing
b. A single elevated acute IgM titer for Mycoplasma and Ureaplasma species.
c. A negative IgM titer followed by two successive IgG 9. Matching: Match each term with the appropriate
titers demonstrating an increase in titer description.
d. A classic paired sera with a definitive fourfold increase ____ M. genitalium a. walking pneumonia
in titer ____ M. fermentans b. crosses placenta after colo-
5. A genital swab was submitted for culture. U. urealyticum ____ U. urealyticum nization of the urogenital
was isolated. The suspected biochemical profile to con- ____ M. pneumoniae tract
firm identification would be: ____ M. hominis c. nongonococcal urethritis,
a. glucose ⫹, arginine ⫹, urease ⫺ cervicitis, and PID
b. glucose ⫺, arginine ⫹, urease ⫹ d. arthritis associated with
c. glucose ⫺, arginine ⫺, urease ⫹ agammaglobulinemia
d. glucose ⫹, arginine ⫺, urease ⫺ e. infections in immunocom-
6. A 21-year-old male presented to his family physician com- promised patients
plaining of flulike symptoms, including a low-grade fever
and body aches. A complete blood count was drawn, and
the patient exhibited no elevation in the white blood cell
count. Serology was negative for influenza. Chest x-ray in-
dicated lobar consolidation representative of a respiratory
infection. Based on the laboratory results and patient pre-
sentation, what is the most likely cause?
a. Influenza A
b. Klebsiella pneumoniae
c. Streptococcus pneumoniae
d. Mycoplasma pneumoniae
e. Pseudomonas aeruginosa
Tille: Bailey & Scott’s Diagnostic Microbiology, 14th Edition
Chapter 44: Cell Wall–Deficient Bacteria: Mycoplasma and Ureaplasma

Answers to Case Studies and Review Questions

Case Studies

Case Study 44-1

1. The patient had an infection with Mycoplasma pneumoniae, which was diagnosed by a
positive test for cold agglutinins. For this test, blood is collected from the patient and
incubated at 4°C. The red blood cells (RBCs) clump, but the clumps disappear at 37°C.
Although this test is not specific for M. pneumoniae, it is positive in approximately 70%
of such infections and is the first test that is positive in the disease. These are oligoclonal
immunoglobulin M (IgM) autoantibodies directed against an altered I antigen on the
surface of the RBCs from infected patients. A titer of 1:128 against type O Rh-negative
erythrocytes is diagnostic. Culture takes several days, and the serum for detection of IgM
antibodies is not positive for approximately 10 days.
2. Generally, bacterial cultures do not include the media needed to detect M. pneumoniae.
Colonies, even on appropriate media, are only microscopically visible. PCR amplification
is available for the detection of M. pneumoniae. However, it is possible to detect the
organism by one method and not the other. The patient may be PCR positive and
serologically and culture negative.
3. M. pneumoniae does not have a cell wall; therefore it lacks the typical peptidoglycan in
the wall of bacteria. All penicillins and cephalosporins act on the cell wall of the bacteria
to inhibit cross-linking of the peptidoglycan. Erythromycin inhibits protein synthesis
inside the bacterial cell. Although M. pneumoniae lacks the cell wall of bacteria, it uses
the same method of protein synthesis shared by other bacteria. Thus erythromycin and
other macrolide antibiotics are effective treatment for infections with this bacterium.

Review Questions

Multiple Choice

1. d; 2. d; 3. e; 4. d 5. c; 6. d; 7. d

True/False

F, T, F, T

Matching

c, e, b, a, d

Copyright © 2017, Elsevier Inc. All Rights Reserved.


CHAPTER 45 The Spirochetes 588.e1

Chapter Review
1. T. pallidum may be transmitted by all of the following 7. A female patient visits her family practice physician and
except: is complaining of severe memory loss as well as muscle
a. Sexual contact and joint pain. She has no additional physical signs or
b. Mother to fetus visible symptoms. What laboratory specimens should the
c. Use of a friend’s toothbrush physician consider collecting for testing and diagnostic
d. None of the above procedures?
2. Secondary syphilis is characterized by: a. Blood, synovial fluid, and CSF
a. Lymphadenopathy, diffuse rash, positive RPR, and b. Blood and urine
negative FTA-ABS c. Urine, synovial fluid, and sputum
b. Lymphadenopathy, weight loss, positive RPR, and d. Only blood
positive FTA-ABS 8. True or False
c. Chancre, weight loss, negative RPR, and positive _____ The presence of antibodies to T. pallidum in a new-
FTA-ABS born child is diagnostic for congenital syphilis.
d. Lymphadenopathy, weight loss, negative RPR, and _____ Leptospirosis can manifest in two primary clinical
positive PaGIA conditions, self-limiting or immune reactions re-
3. Biologic false positives may occur in serologic assays for sulting in nervous system complications.
syphilis as a result of: 9. Matching Match each term with the appropriate definition.
a. Failure to read the reaction at the correct time point _____ Leptospira a. nonspecific antibody
b. Inactivation of the patient’s serum before analysis interrogans b. aseptic meningitis
c. Failure to bring samples and reagents to room _____ Brachyspira spp. c. charcoal and lipid
temperature _____ MHA-TP antigens
d. Pathologic conditions, including other infectious _____ FTA-ABS d. hemagglutination assay
diseases _____ reagin e. neurosyphilis
4. Reagin antibody: _____ cardiolipin f. false brush border
a. Is highly specific for T. pallidum _____ RPR g. CSF specimen
b. Is composed of protein and lipid antigens _____ VDRL h. lipid component
c. Is a nontreponemal lipoidal antibody _____ gumma i. T. pallidum specific
d. Cross-reacts with rheumatoid factor
5. Which organism avoids immune clearance through
genetic variation of outer membrane proteins?
a. T. pallidum
b. Brachyspira spp.
c. Leptospira spp.
d. B. burgdorferi
6. A patient presents to his doctor after a recent deer-hunting
expedition in northern Minnesota. He complains of ab-
dominal discomfort, and he has a red rash on his left arm
and an intermittent fever. What is the most likely cause
of his illness?
a. T. pallidum
b. B. aalborgi
c. B. burgdorferi
d. B. recurrentis
Tille: Bailey & Scott’s Diagnostic Microbiology, 14th Edition
Chapter 45: The Spirochetes

Answers to Case Studies and Review Questions

Case Studies

Case Study 45-1

1. The patient most likely acquired the disease from the bite of the nymphal form of the
Ixodes tick infected with Borrelia burgdorferi. Although the symptoms were not apparent
for a few weeks, her description of the initial lesion is consistent with erythema migrans
(see Figure 46-3).
2. An immunofluorescent assay (IFA) is very sensitive to detect antibodies to B.
burgdorferi, but it is known to have a high false-positive rate. Initially, the patient’s
presentation combined with a positive IFA seemed to be enough to treat the patient
effectively. However, when she did not respond to treatment, the physician wanted to be
certain that the diagnosis was correct. Western blot tests look for a number of antibodies
to the spirochete and are considered positive only if multiple bands are present. This
patient may not have a good serologic response, but the positive PCR was evident that the
organism was still present.
3. Many reasons can explain why patients do not respond to therapy. First, the patient
could have another disease with similar symptoms but one that does not respond to the
therapy for Borrelia. Second, the patient may have a strain that has become resistant to
the antibiotics used to treat the infection. A third reason could be that the dose of
antibiotic was not sufficient to kill the organism. The patient may also have an immune
system that is ineffective in mounting a response to the spirochete. In fact, all of these
reasons could be true for this patient, who may have a poor prognosis.

Review Questions

Multiple Choice

1. c; 2. b; 3. d; 4. c; 5. d; 6. d; 7. a

True/False

F, T

Matching

b, f, d, i, a, h, c, g, e

Copyright © 2017, Elsevier Inc. All Rights Reserved.

You might also like